Final Exam uWu Flashcards

1
Q

Propranolol (Inderal) is prescribed for a patient diagnosed with hypertension. The nurse should consult with the health care provider before giving this medication when the patient reveals a history of

a. asthma.
b. daily alcohol use.
c. peptic ulcer disease.
d. myocardial infarction (MI).

A

a. asthma

How well did you know this?
1
Not at all
2
3
4
5
Perfectly
2
Q

The high-pressure alarm on a patient’s ventilator goes off. When the nurse enters the room to assess the patient, who has acute respiratory distress syndrome (ARDS), the oxygen saturation monitor reads 87% and the patient is struggling to sit up. Which action should the nurse take first?

A) Reassure the patient that the ventilator will do the work of breathing for him.

B) Manually ventilate the patient while assessing possible reasons for the high-pressure alarm.

C) Increase the fraction of inspired oxygen (Fio2) on the ventilator to 100% in preparation for endotracheal suctioning.

D) Insert an oral airway to prevent the patient from biting on the endotracheal tube.

A

B) Manually ventilate the patient while assessing possible reasons for the high-pressure alarm.

Rationale:
Manual ventilation of the patient will allow the nurse to deliver an Fio2 of 100% to the patient while attempting to determine the cause of the high-pressure alarm. The patient may need reassurance, suctioning, or insertion of an oral airway, but the first step should be assessing the reason for the high-pressure alarm and resolving the hypoxemia.

How well did you know this?
1
Not at all
2
3
4
5
Perfectly
3
Q

A patient diagnosed with hypertension has been prescribed captopril. Which information is most important to teach the patient about this drug?

A. Include high-potassium foods such as bananas in the diet
B. Change position slowly to help prevent dizziness and falls
C. Increase fluid intake if dryness of the mouth is a problem
D. Check the blood pressure in both arms before taking the drug

A

B. Change position slowly to help prevent dizziness and falls

How well did you know this?
1
Not at all
2
3
4
5
Perfectly
4
Q

A patient who was admitted with a myocardial infarction experiences a 45-second episode of ventricular tachycardia, then converts to sinus rhythm with a heart rate of 98 beats/minute. Which of the following actions should the nurse take next?

a. Immediately notify the health care provider.
b. Document the rhythm and continue to monitor the patient.
c. Perform synchronized cardioversion per agency dysrhythmia protocol.
d. Prepare to give IV amiodarone (Cordarone) per agency dysrhythmia protocol.

A

D. Prepare to give IV amiodarone (Cordarone) per agency dysrhythmia protocol.

The burst of sustained ventricular tachycardia indicates that the patient has significant ventricular irritability, and antidysrhythmic medication administration is needed to prevent further episodes. The nurse should notify the health care provider after the medication is started. Defibrillation is not indicated given that the patient is currently in a sinus rhythm. Documentation and continued monitoring are not adequate responses to this situation

How well did you know this?
1
Not at all
2
3
4
5
Perfectly
5
Q

A nurse is caring for a patient with ARDS who is being treated with mechanical
ventilation and high levels of positive end-expiratory pressure (PEEP). Which assessment finding by the nurse indicates that the PEEP may need to be reduced?

a. The patient’s PaO2 is 50 mm Hg and the SaO2 is 88%.
b. The patient has subcutaneous emphysema on the upper thorax.
c. The patient has bronchial breath sounds in both the lung fields.
d. The patient has a first-degree atrioventricular heart block with a rate of 58 beats/min.

A

b. The patient has subcutaneous emphysema on the upper thorax.

The subcutaneous emphysema indicates barotrauma caused by positive pressure ventilation and PEEP.
Bradycardia, hypoxemia, and bronchial breath sounds are all concerns and will need to be addressed, but
they are not specific indications that PEEP should be reduced.

How well did you know this?
1
Not at all
2
3
4
5
Perfectly
6
Q

When assessing a patient with ARF, the nurse finds a new onset of agitation and confusion. Which action should the nurse take first?

a. Check pupils for reaction to light.
b. Notify the health care provider.
c. Attempt to calm and reorient the patient.
d. Assess oxygenation using pulse oximetry.

A

d. Assess oxygenation using pulse oximetry.

Because agitation and confusion are frequently the initial indicators of hypoxemia, the nurse’s initial
action should be to assess O2
saturation. The other actions are also appropriate, but assessment of
oxygenation takes priority over other assessments and notification of the health care provider.

How well did you know this?
1
Not at all
2
3
4
5
Perfectly
7
Q

The nurse assesses vital signs for a patient admitted 2 days ago with gram-negative sepsis: temperature of 101.2° F, blood pressure of 90/56 mm Hg, pulse of 92 beats/min, and respirations of 34 breaths/min. Which action should the nurse take next?

a. Give the scheduled IV antibiotic.
b. Give the PRN acetaminophen (Tylenol).
c. Obtain oxygen saturation using pulse oximetry.
d. Notify the health care provider of the patient’s vital signs.

A

c. Obtain oxygen saturation using pulse oximetry.

The patient’s increased respiratory rate in combination with the admission diagnosis of gram-negative sepsis indicates that acute respiratory distress syndrome (ARDS) may be developing. The nurse should
check for hypoxemia, a hallmark of ARDS. The health care provider should be notified after further
assessment of the patient. Giving the scheduled antibiotic and the PRN acetaminophen will also be done,
but they are not the highest priority for a patient who may be developing ARDS.

How well did you know this?
1
Not at all
2
3
4
5
Perfectly
8
Q

The nurse correlates the P wave on the ECG tracing to which cardiac action?

a) Repolarization of the purkinje fibers
b) Repolarization of the ventricles
c) Depolarization of the atria
d) Depolarization of the ventricles

A

c) Depolarization of the atria

How well did you know this?
1
Not at all
2
3
4
5
Perfectly
9
Q

A patient with acute respiratory distress syndrome (ARDS) and acute kidney injury has the following drugs ordered. Which drug should the nurse discuss with the health care provider before giving?

a. gentamicin 60 mg IV
b. pantoprazole (Protonix) 40 mg IV
c. sucralfate (Carafate) 1 g per nasogastric tube
d. methylprednisolone (Solu-Medrol) 60 mg IV

A

a. gentamicin 60 mg IV

Gentamicin, which is one of the aminoglycoside antibiotics, is potentially nephrotoxic, and the nurse should clarify the drug and dosage with the health care provider before administration. The other drugs are appropriate for the patient with ARDS

How well did you know this?
1
Not at all
2
3
4
5
Perfectly
10
Q

When analyzing a patient’s electrocardiogram, the nurse correlates which information as descriptive of a normal P wave?

a. ) Represents conduction through the AV node
b. ) Represents ventricular depolarization
c. ) The height of the P wave is 4 mm
d. ) The P length is 0.10 seconds

A

d.) The P length is 0.10 seconds

How well did you know this?
1
Not at all
2
3
4
5
Perfectly
11
Q

What does the P wave represent?

A

Atrial depolarization

How well did you know this?
1
Not at all
2
3
4
5
Perfectly
12
Q

What does the QRS complex represent?

A

Ventricular depolarization

How well did you know this?
1
Not at all
2
3
4
5
Perfectly
13
Q

What does the T wave represent?

A

Ventricular repolarization

How well did you know this?
1
Not at all
2
3
4
5
Perfectly
14
Q

A patient is being evaluated for a blockage in the cardiac ventricles. The nurse assesses which part of the ECG as evidence of this blockage?

a. ) T wave
b. ) U wave
c. ) PR interval
d. ) QRS interval

A

d.) QRS interval

How well did you know this?
1
Not at all
2
3
4
5
Perfectly
15
Q

The nurse prepares to administer which prescribed medication to the patient with shortness of breath and a heart rhythm of 46 beats per minute?

a. ) Atropine sulfate
b. ) Atenolol
c. ) Diltiazem
d. ) Adenosine

A

a.) Atropine sulfate

How well did you know this?
1
Not at all
2
3
4
5
Perfectly
16
Q

The nurse provides care to a patient who is prescribed IV adenosine. Which is the priority nursing action before administering the medication?

a. ) Monitoring respirations
b. ) Palpating the patient’s pulse
c. ) Teaching the patient to avoid caffeine intake
d. ) Ensuring a transcutaneous pacemaker is available

A

d.) Ensuring a transcutaneous pacemaker is available

How well did you know this?
1
Not at all
2
3
4
5
Perfectly
17
Q

The nurse is preparing to defibrillate a patient and selects which setting before the first shock?

a. ) 50 J
b. ) 100 J
c. ) 150 J
d. ) 200 J

A

d.) 200 J

How well did you know this?
1
Not at all
2
3
4
5
Perfectly
18
Q

Once ventricular fibrillation has been confirmed in a patient, which action is the priority?

a. ) Assessing vital signs
b. ) Opening the airway
c. ) Beginning rescue breathing
d. ) Starting chest compressions
d. ) Starting chest compressions

A

d.) Starting chest compressions

How well did you know this?
1
Not at all
2
3
4
5
Perfectly
19
Q

A patient becomes unresponsive without a palpable pulse despite showing bradycardia on the rhythm strip. What action should the nurse take immediately? (Select all that apply)

a. ) Auscultating heart sounds
b. ) Beginning cardiac compressions
c. ) Adjusting cardiac monitor leads
d. ) Obtaining blood samples for electrolytes
e. ) Placing epinephrine at the bedside

A

b. ) Beginning cardiac compressions
e. ) Placing epinephrine at the bedside

How well did you know this?
1
Not at all
2
3
4
5
Perfectly
20
Q

What is the order of electrical impulse through the heart?

A

1) SA node fires
2) Impulse spreads through the atrial myocardium
3) Impulse travels to the AV node
4) Impulse leaves the AV node through the bundle of His
5) Impulse travels through the bundle branches
6) Impulse extends into the ventricular tissue through the purkinje fibers

How well did you know this?
1
Not at all
2
3
4
5
Perfectly
21
Q

What are some major risk factors for cardiovascular disease?

A

Family history of cardiovascular disease
Diabetes mellitus
Chronic renal disease
Hypertension
Dyslipidemia

How well did you know this?
1
Not at all
2
3
4
5
Perfectly
22
Q

What are some examples of modifiable risk factors?

A

Weight
Dietary habits
Alcohol consumption
Smoking

How well did you know this?
1
Not at all
2
3
4
5
Perfectly
23
Q

The nurse is performing an assessment on a client and comes up with a blood pressure reading of 140/95. The client then states, “my blood pressure has been higher than that the last two times I have checked it”. Upon further evaluation, the nurse finds this to be true and knows that the client is experiencing what?

a) Stage 1 hypertension
b) Stage 2 hypertension
c) Nothing, this blood pressure reading is normal.

A

b) Stage 2 hypertension

How well did you know this?
1
Not at all
2
3
4
5
Perfectly
24
Q

What is the purpose of a transesophageal echocardiogram (TEE)?

A

This exam can allow doctors to get a clearer and better picture of the heart. This is most commonly used to see specific structures of the heart or to visualize clots in the heart chambers.

How well did you know this?
1
Not at all
2
3
4
5
Perfectly
25
Q

An older adult client is being evaluated for a new onset of a cardiac dysrhythmia. What should the nurse consider as being the cause for this abnormal heart rhythm? (Select all that apply)

a. ) Advanced age
b. ) Protein malnutrition
c. ) Fat deposits around the sinoatrial node
d. ) Fewer pacemaker cells in the sinoatrial node
e. ) Calcification around the atrioventricular node and valves

A

a. ) Advanced age
c. ) Fat deposits around the sinoatrial node
d. ) Fewer pacemaker cells in the sinoatrial node
e. ) Calcification around the atrioventricular node and valves

How well did you know this?
1
Not at all
2
3
4
5
Perfectly
26
Q

The nurse provides care to a patient who is prescribed IV adenosine. Which is the priority nursing action before administering the medication?

a. ) Monitoring respirations
b. ) Palpating the patient’s pulse
c. ) Teaching the patient to avoid caffeine intake
d. ) Ensuring a transcutaneous pacemaker is available

A

d.) Ensuring a transcutaneous pacemaker is available

How well did you know this?
1
Not at all
2
3
4
5
Perfectly
27
Q

In preparing a patient with atrial fibrillation for cardioversion, the nurse prepares the patient for which diagnostic test prior?

a. ) Chest x-ray
b. ) CT scan of the chest
c. ) 12-lead ECG
d. ) Transesophageal echocardiogram (TEE)

A

d.) Transesophageal echocardiogram (TEE)

How well did you know this?
1
Not at all
2
3
4
5
Perfectly
28
Q

A patient is being cardioverted for symptomatic atrial fibrillation. At which point of the cardiac cycle will the electric impulse occur?

a. ) At the end of the P wave
b. ) Before the QRS complex
c. ) At the peak of the R wave
d. ) After the QRS complex but before the T wave

A

c.) At the peak of the R wave

How well did you know this?
1
Not at all
2
3
4
5
Perfectly
29
Q

The nurse monitors for which clinical manifestation in the patient with atrial fibrillation at a heart rate of 90 beats per minute. Which manifestation should the nurse expect to assess in this patient?

a. ) Headache
b. ) Chest pain
c. ) Palpitations
d. ) Hypotension

A

c.) Palpitations

How well did you know this?
1
Not at all
2
3
4
5
Perfectly
30
Q

The nurse determines that demeclocycline (Declomycin) is effective for a patient with syndrome of inappropriate antidiuretic hormone (SIADH) based on finding that the patient’s

a. weight has increased.
b. urinary output is increased.
c. peripheral edema is decreased.
d. urine specific gravity is increased.

A

b. urinary output is increased.

Demeclocycline blocks the action of antidiuretic hormone (ADH) on the renal tubules and increases urine output. An increase in weight or an increase in urine specific gravity indicates that the SIADH is not corrected. Peripheral edema does not occur with SIADH. A sudden weight gain without edema is a common clinical manifestation of this disorder.

How well did you know this?
1
Not at all
2
3
4
5
Perfectly
31
Q

The nurse determines that additional instruction is needed for a 60-year-old patient with chronic syndrome of inappropriate antidiuretic hormone (SIADH) when the patient says which of the following?

a. “I need to shop for foods low in sodium and avoid adding salt to food.”
b. “I should weigh myself daily and report any sudden weight loss or gain.”
c. “I need to limit my fluid intake to no more than 1 quart of liquids a day.”
d. “I will eat foods high in potassium because diuretics cause potassium loss.”

A

a. “I need to shop for foods low in sodium and avoid adding salt to food.”

Patients with SIADH are at risk for hyponatremia, and a sodium supplement may be prescribed. The other patient statements are correct and indicate successful teaching has occurred

How well did you know this?
1
Not at all
2
3
4
5
Perfectly
32
Q

A 56-year-old patient who is disoriented and reports a headache and muscle cramps is hospitalized with possible syndrome of inappropriate antidiuretic hormone (SIADH). The nurse would expect the initial laboratory results to include a(n)

a. elevated hematocrit.
b. decreased serum sodium.
c. low urine specific gravity.
d. increased serum chloride.

A

b. decreased serum sodium.

When water is retained, the serum sodium level will drop below normal, causing the clinical manifestations reported by the patient. The hematocrit will decrease because of the dilution caused by water retention. Urine will be more concentrated with a higher specific gravity. The serum chloride level will usually decrease along with the sodium level.

How well did you know this?
1
Not at all
2
3
4
5
Perfectly
33
Q

Which intervention will the nurse include in the plan of care for a 52-year-old male patient with syndrome of inappropriate antidiuretic hormone (SIADH)?

a. Monitor for peripheral edema.
b. Offer patient hard candies to suck on.
c. Encourage fluids to 2 to 3 liters per day.
d. Keep head of bed elevated to 30 degrees.

A

b. Offer patient hard candies to suck on.

Sucking on hard candies decreases thirst for a patient on fluid restriction. Patients with SIADH are on fluid restrictions of 800 to 1000 mL/day. Peripheral edema is not seen with SIADH. The head of the bed is elevated no more than 10 degrees to increase left atrial filling pressure and decrease antidiuretic hormone (ADH) release.

How well did you know this?
1
Not at all
2
3
4
5
Perfectly
34
Q

Which information is most important for the nurse to communicate rapidly to the health care provider about a patient admitted with possible syndrome of inappropriate antidiuretic hormone (SIADH)?

a. The patient has a recent weight gain of 9 lb.
b. The patient complains of dyspnea with activity.
c. The patient has a urine specific gravity of 1.025.
d. The patient has a serum sodium level of 118 mEq/L.

A

d. The patient has a serum sodium level of 118 mEq/L.

A serum sodium of less than 120 mEq/L increases the risk for complications such as seizures and needs rapid correction. The other data are not unusual for a patient with SIADH and do not indicate the need for rapid action

How well did you know this?
1
Not at all
2
3
4
5
Perfectly
35
Q

Which action should the nurse take when administering the initial dose of oral labetalol (Normodyne) to a patient with hypertension?

a. Encourage the use of hard candy to prevent dry mouth.
b. Instruct the patient to ask for help if heart palpitations occur.
c. Ask the patient to request assistance when getting out of bed.
d. Teach the patient that headaches may occur with this medication.

A

c. Ask the patient to request assistance when getting out of bed.

How well did you know this?
1
Not at all
2
3
4
5
Perfectly
36
Q

After the nurse teaches the patient with stage 1 hypertension about diet modifications that should be implemented, which diet choice indicates that the teaching has been effective?

a. The patient avoids eating nuts or nut butters.
b. The patient restricts intake of chicken and fish.
c. The patient has two cups of coffee in the morning.
d. The patient has a glass of low-fat milk with each meal.

A

d. The patient has a glass of low-fat milk with each meal.

How well did you know this?
1
Not at all
2
3
4
5
Perfectly
37
Q

An older patient has been diagnosed with possible white coat hypertension. Which action will the nurse plan to take next?

a. Schedule the patient for regular blood pressure (BP) checks in the clinic.
b. Instruct the patient about the need to decrease stress levels.
c. Tell the patient how to self-monitor and record BPs at home.
d. Inform the patient that ambulatory blood pressure monitoring will be needed.

A

c. Tell the patient how to self-monitor and record BPs at home.

How well did you know this?
1
Not at all
2
3
4
5
Perfectly
38
Q

The nurse is reviewing the laboratory test results for a patient who has recently been diagnosed with hypertension. Which result is most important to communicate to the health care provider?

a. Serum creatinine of 2.8 mg/dL
b. Serum potassium of 4.5 mEq/L
c. Serum hemoglobin of 14.7 g/dL
d. Blood glucose level of 96 mg/dL

A

a. Serum creatinine of 2.8 mg/dL

How well did you know this?
1
Not at all
2
3
4
5
Perfectly
39
Q

A patient with a history of hypertension treated with a diuretic and an angiotensin-converting enzyme (ACE) inhibitor arrives in the emergency department complaining of a severe headache and nausea and has a blood pressure (BP) of 238/118 mm Hg. Which question should the nurse ask first?

a. “Did you take any acetaminophen (Tylenol) today?”
b. “Have you been consistently taking your medications?”
c. “Have there been any recent stressful events in your life?”
d. “Have you recently taken any antihistamine medications?”

A

b. “Have you been consistently taking your medications?”

How well did you know this?
1
Not at all
2
3
4
5
Perfectly
40
Q

When reviewing the 12-lead electrocardiograph (ECG) for a healthy 86-year-old patient who is having an annual physical examination, which of the following will be of most concern to the nurse?

a. The heart rate (HR) is 43 beats/minute.
b. The PR interval is 0.21 seconds.
c. There is a right bundle-branch block.
d. The QRS duration is 0.13 seconds

A

a. The heart rate (HR) is 43 beats/minute.

How well did you know this?
1
Not at all
2
3
4
5
Perfectly
41
Q

A patient is scheduled for a cardiac catheterization with coronary angiography. Before the test, the nurse informs the patient that

a. electrocardiographic (ECG) monitoring will be required for 24 hours after the test.
b. it will be important to lie completely still during the procedure.
c. a warm feeling may be noted when the contrast dye is injected.
d. monitored anesthesia care will be provided during the procedure.

A

c. a warm feeling may be noted when the contrast dye is injected.

How well did you know this?
1
Not at all
2
3
4
5
Perfectly
42
Q

The nurse expects that management of the patient who experiences a brief episode of tinnitus, diplopia, and dysarthria with no residual effects will include

a. prophylactic clipping of cerebral aneurysms.
b. heparin via continuous intravenous infusion.
c. oral administration of low dose aspirin therapy.
d. therapy with tissue plasminogen activator (tPA).

A

c. oral administration of low dose aspirin therapy.

How well did you know this?
1
Not at all
2
3
4
5
Perfectly
43
Q

The nurse identifies the nursing diagnosis of impaired verbal communication for a patient with expressive aphasia. An appropriate nursing intervention to help the patient communicate is to

a. have the patient practice facial and tongue exercises.
b. ask simple questions that the patient can answer with “yes” or “no.”
c. develop a list of words that the patient can read and practice reciting.
d. prevent embarrassing the patient by changing the subject if the patient does not respond.

A

b. ask simple questions that the patient can answer with “yes” or “no.”

How well did you know this?
1
Not at all
2
3
4
5
Perfectly
44
Q

A patient has a stroke affecting the right hemisphere of the brain. Based on knowledge of the effects of right brain damage, the nurse establishes a nursing diagnosis of

a. impaired physical mobility related to right hemiplegia.
b. risk for injury related to denial of deficits and impulsiveness.
c. impaired verbal communication related to speech-language deficits.
d. ineffective coping related to depression and distress about disability.

A

b. risk for injury related to denial of deficits and impulsiveness.

How well did you know this?
1
Not at all
2
3
4
5
Perfectly
45
Q

When caring for a patient with left-sided homonymous hemianopsia resulting from a stroke, which intervention should the nurse include in the plan of care during the acute period of the stroke?

a. Apply an eye patch to the left eye.
b. Approach the patient from the left side.
c. Place objects needed for activities of daily living on the patient’s right side.
d. Reassure the patient that the visual deficit will resolve as the stroke progresses.

A

c. Place objects needed for activities of daily living on the patient’s right side.

How well did you know this?
1
Not at all
2
3
4
5
Perfectly
46
Q

The nurse identifies the nursing diagnosis of imbalanced nutrition: less than body requirements related to impaired self-feeding ability for a patient with right-sided hemiplegia. Which intervention should be included in the plan of care?

a. Provide a wide variety of food choices.
b. Provide oral care before and after meals.
c. Assist the patient to eat with the left hand.
d. Teach the patient the “chin-tuck” technique.

A

c. Assist the patient to eat with the left hand.

How well did you know this?
1
Not at all
2
3
4
5
Perfectly
47
Q

A 32-year-old patient has a stroke resulting from a ruptured aneurysm and subarachnoid hemorrhage. Which intervention will be included in the care plan?

a. Applying intermittent pneumatic compression stockings
b. Assisting to dangle on edge of bed and assess for dizziness
c. Encouraging patient to cough and deep breathe every 4 hours
d. Inserting an oropharyngeal airway to prevent airway obstruction

A

a. Applying intermittent pneumatic compression stockings

How well did you know this?
1
Not at all
2
3
4
5
Perfectly
48
Q

Several weeks after a stroke, a patient has urinary incontinence resulting from an impaired awareness of bladder fullness. For an effective bladder training program, which nursing intervention will be best to include in the plan of care?

a. Limit fluid intake to 1200 mL daily to reduce urine volume.
b. Assist the patient onto the bedside commode every 2 hours.
c. Perform intermittent catheterization after each voiding to check for residual urine.
d. Use an external “condom” catheter to protect the skin and prevent embarrassment.

A

b. Assist the patient onto the bedside commode every 2 hours.

How well did you know this?
1
Not at all
2
3
4
5
Perfectly
49
Q

A patient with chronic atrial fibrillation develops sudden severe pain, pulselessness, pallor, and coolness in the left leg. The nurse should notify the health care provider and

a. elevate the left leg on a pillow.
b. apply an elastic wrap to the leg.
c. assist the patient in gently exercising the leg.
d. keep the patient in bed in the supine position

A

d. keep the patient in bed in the supine position

How well did you know this?
1
Not at all
2
3
4
5
Perfectly
50
Q

A patient at the clinic says, “I have always taken an evening walk, but lately my leg cramps and hurts after just a few minutes of walking. The pain goes away after I stop walking, though.” The nurse should

a. attempt to palpate the dorsalis pedis and posterior tibial pulses.
b. check for the presence of tortuous veins bilaterally on the legs.
c. ask about any skin color changes that occur in response to cold.
d. assess for unilateral swelling, redness, and tenderness of either leg.

A

a. attempt to palpate the dorsalis pedis and posterior tibial pulses.

How well did you know this?
1
Not at all
2
3
4
5
Perfectly
51
Q

The nurse has initiated discharge teaching for a patient who is to be maintained on warfarin (Coumadin) following hospitalization for venous thromboembolism (VTE). The nurse determines that additional teaching is needed when the patient says,

a. “I should reduce the amount of green, leafy vegetables that I eat.”
b. “I should wear a Medic Alert bracelet stating that I take Coumadin.”
c. “I will need to have blood tests routinely to monitor the effects of the Coumadin.”
d. “I will check with my health care provider before I begin or stop any medication.”

A

a. “I should reduce the amount of green, leafy vegetables that I eat.”

How well did you know this?
1
Not at all
2
3
4
5
Perfectly
52
Q

What is depolarization?

What is repolarization?

A

Depolarization - Contraction of the heart

Repolarization - Resting state of the heart

How well did you know this?
1
Not at all
2
3
4
5
Perfectly
53
Q

A 62-year old man with chronic anemia is experiencing increased fatigue and occasional palpitations at rest. The nurse would expect the patient’s laboratory findings to include

a. a hematocrit (Hct) of 38%.
b. an RBC count of 4,500,000/μL.
c. normal red blood cell (RBC) indices.
d. a hemoglobin (Hgb) of 8.6 g/dL (86 g/L)

A

d. a hemoglobin (Hgb) of 8.6 g/dL (86 g/L)

How well did you know this?
1
Not at all
2
3
4
5
Perfectly
54
Q

A patient who is receiving methotrexate for severe rheumatoid arthritis develops a megaloblastic anemia. The nurse will anticipate teaching the patient about increasing oral intake of

a. iron.
b. folic acid.
c. cobalamin (vitamin B12).
d. ascorbic acid (vitamin C).

A

b. folic acid.

How well did you know this?
1
Not at all
2
3
4
5
Perfectly
55
Q

On admission of a patient to the postanesthesia care unit (PACU), the blood pressure (BP) is 122/72. Thirty minutes after admission, the BP falls to 114/62, with a pulse of 74 and warm, dry skin. Which action by the nurse is most appropriate?

a. Increase the IV fluid rate.
b. Continue to take vital signs every 15 minutes.
c. Administer oxygen therapy at 100% per mask.
d. Notify the anesthesia care provider (ACP) immediately.

A

b. Continue to take vital signs every 15 minutes.

How well did you know this?
1
Not at all
2
3
4
5
Perfectly
56
Q

The nurse describes to a student nurse how to use evidence­based practice guidelines when caring for patients. Which statement, if made by the nurse, would be the most accurate?

a. “Inferences from clinical research studies are used as a guide.”
b. “Patient care is based on clinical judgment, experience, and traditions.”
c. “Data are evaluated to show that the patient outcomes are consistently met.”
d. “Recommendations are based on research, clinical expertise, and patient preferences.”

A

d. “Recommendations are based on research, clinical expertise, and patient preferences.”

How well did you know this?
1
Not at all
2
3
4
5
Perfectly
57
Q

A nurse is caring for a patient with heart failure. Which task is appropriate for the nurse to delegate to experienced unlicensed assistive personnel (UAP)?

a. Monitor for shortness of breath or fatigue after ambulation.
b. Instruct the patient about the need to alternate activity and rest.
c. Obtain the patient’s blood pressure and pulse rate after ambulation.
d. Determine whether the patient is ready to increase the activity level.

A

c. Obtain the patient’s blood pressure and pulse rate after ambulation.

How well did you know this?
1
Not at all
2
3
4
5
Perfectly
58
Q

A patient with newly diagnosed type 2 diabetes mellitus asks the nurse what ““type 2”” means in relation to diabetes. The nurse explains to the patient that type 2 diabetes differs from type 1 diabetes primarily in that with type 2 diabetes

a. the pt is totally dependent on an outside source of insulin
b. there is a decreased insulin secretion and cellular resistance to insulin that is produced
c. the immune system destroys the pancreatic insulin-producing cells
d. the insulin precurosr that is secreted by the pancreas is not activated by the liver

A

b. there is a decreased insulin secretion and cellular resistance to insulin that is produced

In type 2 diabetes, the pancreas produces insulin, but the insulin is insufficient for the body’s needs or the cells do not respond to the insulin appropriately. The other information describes the physiology of type 1 diabetes

How well did you know this?
1
Not at all
2
3
4
5
Perfectly
59
Q

The benefits of using an insulin pump include all of the following except: “

a. By continuously providing insulin they eliminate the need for injections of insulin
b. They simplify management of blood sugar and often improve A1C
c. They enable exercise without compensatory carbohydrate consumption
d. They help with weight loss

A

d. They help with weight loss

Using an insulin pump has many advantages, including fewer dramatic swings in blood glucose levels, increased flexibility about diet, and improved accuracy of insulin doses and delivery; however, the use of an insulin pump has been associated with weight gain.

How well did you know this?
1
Not at all
2
3
4
5
Perfectly
60
Q

A 54-year-old patient admitted with type 2 diabetes, asks the nurse what “type 2” means. Which of the following is the most appropriate response by the nurse?

“1. ““With type 2 diabetes, the body of the pancreas becomes inflamed.”

  1. “With type 2 diabetes, insulin secretion is decreased and insulin resistance is increased.”
  2. “With type 2 diabetes, the patient is totally dependent on an outside source of insulin.”
  3. “With type 2 diabetes, the body produces autoantibodies that destroy b-cells in the pancreas.””
A
  1. “With type 2 diabetes, insulin secretion is decreased and insulin resistance is increased.”

Rationale: In type 2 diabetes mellitus, the secretion of insulin by the pancreas is reduced and/or the cells of the body become resistant to insulin”

How well did you know this?
1
Not at all
2
3
4
5
Perfectly
61
Q

“A client is admitted to the hospital with signs and symptoms of diabetes mellitus. Which findings is the nurse most likely to observe in this client? Select all that apply:

  1. Excessive thirst
  2. Weight gain
  3. Constipation
  4. Excessive hunger
  5. Urine retention
  6. Frequent, high-volume urination
A
  1. Excessive thirst
  2. Excessive hunger
  3. Frequent, high-volume urination

Rationale: Classic signs of diabetes mellitus include polydipsia (excessive thirst), polyphagia (excessive hunger), and polyuria (excessive urination). Because the body is starving from the lack of glucose the cells are using for energy, the client has weight loss, not weight gain. Clients with diabetes mellitus usually don’t present with constipation. Urine retention is only a problem is the patient has another renal-related condition.

How well did you know this?
1
Not at all
2
3
4
5
Perfectly
62
Q

A client is brought to the emergency department in an unresponsive state, and a diagnosis of hyperglycemic hyperosmolar nonketotic syndrome is made. The nurse would immediately prepare to initiate which of the following anticipated physician’s prescriptions?

  1. Endotracheal intubation
  2. 100 units of NPH insulin
  3. Intravenous infusion of normal saline
  4. Intravenous infusion of sodium bicarbonate
A
  1. Intravenous infusion of normal saline

Intravenous infusion of normal saline Rationale: The primary goal of treatment is hyperglycemic hyperosmolar nonketotic syndrome (HHNS) is to rehydrate the client to restore the fluid volume and to correct electrolyte deficiency. Intravenous fluid replacement is similar to that administered in diabetic keto acidosis (DKA) and begins with IV infusion of normal saline. Regular insulin, not NPH insulin, would be administered. The use of sodium bicarbonate to correct acidosis is avoided because it can precipitate a further drop in serum potassium levels. Intubation and mechanical ventilation are not required to treat HHNS.

How well did you know this?
1
Not at all
2
3
4
5
Perfectly
63
Q

“A client is taking Humulin NPH insulin daily every morning. The nurse instructs the client that the mostlikely time for a hypoglycemic reaction to occur is:

A) 2-4 hours after administration

B) 4-12 hours after administration

C) 16-18 hours after administration

D) 18-24 hours after administration

A

B) 4-12 hours after administration

Rationale: Humulin is an intermediate acting insulin. The onset of action is 1.5 hours, it peaks in 4-12 hours, and its duration is 24 hours. Hypoglycemic reactions to insulin are most likely to occur during the peak time.

How well did you know this?
1
Not at all
2
3
4
5
Perfectly
64
Q

“A client who is started on metformin and glyburide would have initially presented with which symptoms? “

a. Polydispisa, polyuria, and weight loss
b. weight gain, tiredness, & bradycardia
c. irritability, diaphoresis, and tachycardia
d. diarrhea, abdominal pain, and weight loss

A

a. Polydispisa, polyuria, and weight loss

Symptoms of hyperglycemia include polydipsia, polyuria, and weight loss. Metformin and sulfonylureas are commonly ordered medications. Weight gain, tiredness, and bradycardia are symptoms of hypothyroidism. Irritability, diaphoresis, and tachycardia are symptoms of hypoglycemia. Symptoms of Crohn’s disease include diarrhea, abdominal pain, and weight loss.”

How well did you know this?
1
Not at all
2
3
4
5
Perfectly
65
Q

A client with diabetes mellitus demonstratees acute anxiety when first admitted for the treatment of hyperglycemia. The most appropriate intervention to decrease the client’s anxiety would be to

  1. administer a sedative
  2. make sure the client knows all the correct medical terms to understand what is happening
  3. ignore the signs and symptoms of anxiety so that they will soon disappear
  4. convey empathy, trust, and respect toward the client
A
  1. convey empathy, trust, and respect toward the client

The most appropriate intervention is to address the client’s feelings related to the anxiety

How well did you know this?
1
Not at all
2
3
4
5
Perfectly
66
Q

A client with diabetes melllitus has a blood glucose of 644mg/dl. The nurse intreprets that this client is most at risk of developing which type of acid base imbalance? “

A. Metabolic acidosis

B. Metabolic alkalosis

C. Respiratory Acidosis

D. Respiratory Alkalosis”

A

A. Metabolic acidosis

Rationale: DM can lead to metabolic acidosis. When the body does not have sufficient circulating insulin, the blood glucose level rises. At the same time, the cells of the body use all available glucose. The body then breaks down glycogen and fat for fuel. The by-products of fat metabolism are acidotic and can lead to the condition known as diabetic ketoacidosis.”

How well did you know this?
1
Not at all
2
3
4
5
Perfectly
67
Q

A client with DKA is being treated in the ED. What would the nurse suspect?

  1. Comatose state
  2. Decreased Urine Output
  3. Increased respirations and an increase in pH.
  4. Elevated blood glucose level and low plasma bicarbonate level.
A
  1. Elevated blood glucose level and low plasma bicarbonate level.

Rationale: In DKA the arteriole pH is lower than 7.35, plasma bicarbonate is lower than 15 mEq/L, the blood glucose is higher than 250, and ketones are present in the blood and urine. The client would be experiencing polyuria and Kussmauls respirations would be present. A comatose state may occur if DKA is not treated.

How well did you know this?
1
Not at all
2
3
4
5
Perfectly
68
Q

A client with type I diabetes is placed on an insulin pump. The most appropriate short-term goal when teaching this client to control the diabetes is: “

1) adhere to the medical regimen
2) remain normoglycemic for 3 weeks
3) demonstrate the correct use of the administration equipment.
4) list 3 self care activities that are necessary to control the diabetes”

A

CORRECT 3) demonstrate the correct use of the administration equipment.

1) this is not a short-term goal
2) this is measurable, but it’s a long-term goal
3) this is a short-term goal, client oriented, necessary for the client to control the diabetes, and measurable when the client performs a return demonstration for the nurse
4) although this is measurable and a short-term goal, it is not the one with the greatest priority when a client has an insulin pump that must be mastered before discharge”

How well did you know this?
1
Not at all
2
3
4
5
Perfectly
69
Q

A diabetic patient has a serum glucose level of 824 mg/dL (45.7 mmol/L) and is unresponsive. Following assessment of the patient, the nurse suspects diabetic ketoacidosis rather than hyperosmolar hyperglycemic syndrome based on the finding of “

a. polyuria
b. severe dehydration
c. rapid, deep respirations
d. decreased serum potassium”

A

c. rapid, deep respirations

is correct, Signs and symptoms of DKA include manifestations of dehydration such as poor skin turgor, dry mucous membranes, tachycardia, and orthostatic hypotension. Early symptoms may include lethargy and weakness. As the patient becomes severely dehydrated, the skin becomes dry and loose, and the eyeballs become soft and sunken. Abdominal pain is another symptom of DKA that may be accompanied by anorexia and vomiting. Kussmaul respirations (i.e., rapid, deep breathing associated with dyspnea) are the body’s attempt to reverse metabolic acidosis through the exhalation of excess carbon dioxide. Acetone is identified on the breath as a sweet, fruity odor. Laboratory findings include a blood glucose level greater than 250 mg/dL, arterial blood pH less than 7.30, serum bicarbonate level less than 15 mEq/L, and moderate to large ketone levels in the urine or blood ketones.

How well did you know this?
1
Not at all
2
3
4
5
Perfectly
70
Q

A frail elderly patient with a diagnosis of type 2 diabetes mellitus has been ill with pneumonia. The cliet’s intake has been very poor, and she is admitted to the hospital for observation and management as needed. What is the most likely problem with this patient? “

A. Insulin resistance has developed.

B. Diabetic ketoacidosis is occuring.

C. Hypoglycemia unawareness is developing.

D. Hyperglycemic hyperosmolar non-ketotic coma.

A

D. Hyperglycemic hyperosmolar non-ketotic coma.

Illness, especially with the frail elderly patient whose appetite is poor, can result in dehydration and HHNC. Insulin resisitance is inidcated by a daily insulin requirement of 200 units or more. Diabetic ketoacidosis, an acute metabolic condition, usually is caused by absent or markedly decreased amounts of insulin.

How well did you know this?
1
Not at all
2
3
4
5
Perfectly
71
Q

A home health nurse is at the home of a client with diabetes and arthritis. The client has difficulty drawing up insulin. It would be most appropriate for the nurse to refer the client to: “

A) A social worker from the local hospital

B) An occupational therapist from the community center

C) A physical therapist from the rehabilitation agency

D) Another client with diabetes mellitus and takes insulin”

A

B) An occupational therapist from the community center

An occupational therapist can assist a client to improve the fine motor skills needed to prepare an insulin injection.

How well did you know this?
1
Not at all
2
3
4
5
Perfectly
72
Q

A nurse is caring for a cient with type 1 diabetes mellitus. which client complaint would alert the nurse to the presence of a possible hypoglycemic reaction? “

  1. Tremors
  2. Anorexia
  3. Hot, dry skin
  4. Muscle cramps
A
  1. Tremors

decreased blood glucose levels produce autonomic nervous system symptoms, which are manifested classically as nervousness, irritability, and tremors. option 3 is more likely for hyperglycemia, and options 2 and 4 are unrelated to the signs of hypoglycemia.

How well did you know this?
1
Not at all
2
3
4
5
Perfectly
73
Q

A nurse is caring for a client admitted to the emergency department with diabetic ketoacidosis (DKA). In the acute phase, the priority nursing action is to prepare to:

A. Correct the acidosis

B. Administer 5% dextrose intravenously

C. Administer regular insulin inraVenously

D. Apply a monitor for an electrocardiogram.

A

C. Administer regular insulin inraVenously

Lack (absolute or relative) of insulin is the primary cause of DKA. Treatment consists of insulin administration (regular insulin), intravenous fluid administration (normal saline initially), and potassium replacement, followed by correcting acidosis. Applying an electrocardiogram monitor is not a priority action.

How well did you know this?
1
Not at all
2
3
4
5
Perfectly
74
Q

a nurse is interviewing a client with type 2 diabetes mellitus. which statement by the client indicated an understanding of the treatment for this disorder? “

  1. "”i take oral insulin instead of shots””
  2. "”by taking these medications I am able to eat more””
  3. "”when I become ill, I need to increase the number of pills I take””
  4. "”the medications I’m taking help release the insulin I already make”
A
  1. “the medications I’m taking help release the insulin I already make”

Clients with type 2 diabetes mellitus have decreased or imparied insulin secretion. Oral hypoglycemic agents are given to these clients to facilitate glucose uptake. Insulin injections may be given during times of stress-induced hyperglycemia. Oral insulin is not available because of the breakdown of the insulin by digestion. Options 1, 2 and 3 are incorrect

How well did you know this?
1
Not at all
2
3
4
5
Perfectly
75
Q

A nurse is preparing a teaching plan for a client with diabetes Mellitus regarding proper foot care. Which instruction is included in the plan?

  1. Soak feet in hot water
  2. apply a moisturizing lotion to dry feet but not between the toes
  3. Always have a podiatrist cut your toenails, never cut them yourself
  4. avoid using mild soap on the feet
A
  1. apply a moisturizing lotion to dry feet but not between the toes

The client is instructed to use a moisturizing lotion on the feet and to avoid applying the lotion between the toes.

How well did you know this?
1
Not at all
2
3
4
5
Perfectly
76
Q

A nurse performs a physical assessment on a client with type 2 diabetes mellitus. Findings include a fasting blood glucose of 120 mg/dL, temp of 101 F, pulse of 88 bpm, respirations of 22, and blood pressure of 100/72. Which finding would be of most concern to the nurse?

  1. Pulse
  2. Respiration
  3. Temperature
  4. Blood pressure
A
  1. Temperature

An elevated temperature may indicate infection. Infection is a leading cause of hyperglycemic hyperosmolar nonketotic syndrome or diabetic ketoacidosis. The other findings noted in the question are within normal limits.

How well did you know this?
1
Not at all
2
3
4
5
Perfectly
77
Q

A nurse shoud recognize which symptom as a cardinal sign of diabetes mellitus? “

a. Nausea
b. Seizure
c. Hyperactivity
d. Frequent urination

A

d. Frequent urination

Polyphagia, polyuria, polydipsia, and weight loss are cardinal signs of DM. Other signs include irritability, shortened attention span, lowered frustration tolerance, fatigue, dry skin, blurred vision, sores that are slow to heal, and flushed skin.

How well did you know this?
1
Not at all
2
3
4
5
Perfectly
78
Q

A patient is admitted with diabetes mellitus, has a glucose level of 380 mg/dl, and a moderate level of ketones in the urine. As the nurse assesses for signs of ketoacidosis, which of the following respiratory patterns would the nurse expect to find?”

A-Central apnea

B-Hypoventilation

C-Kussmaul respirations

D- Cheyne-Stokes respirations”

A

C-Kussmaul respirations

In diabetic ketoacidosis, the lungs try to compensate for the acidosis by blowing off volatile acids and carbon dioxide. This leads to a pattern of Kussmaul respirations, which are deep and nonlabored.

How well did you know this?
1
Not at all
2
3
4
5
Perfectly
79
Q

“A patient with type 1 diabetes has received diet instruction as part of the treatment plan. The nurse determines a need for additional instruction when the patient says, “

a. ““I may have an occasional alcoholic drink if I include it in my meal plan.””
b. ““I will need a bedtime snack because I take an evening dose of NPH insulin.””
c. ““I will eat meals as scheduled, even if I am not hungry, to prevent hypoglycemia.””
d. ““I may eat whatever I want, as long as I use enough insulin to cover the calories.

A

D. ““I may eat whatever I want, as long as I use enough insulin to cover the calories.””

Rationale: Most patients with type 1 diabetes need to plan diet choices very carefully. Patients who are using intensified insulin therapy have considerable flexibility in diet choices but still should restrict dietary intake of items such as fat, protein, and alcohol. The other patient statements are correct and indicate good understanding of the diet instruction.”

How well did you know this?
1
Not at all
2
3
4
5
Perfectly
80
Q

“An 18-year-old female client, 5’4’’ tall, weighing 113 kg, comes to the clinic for a non-healing wound on her lower leg, which she has had for two weeks. Which disease process should the nurse suspect the client is developing? “

A. Type 1 diabetes

B. Type 2 diabetes

C. Gestational diabetes

D. Acanthosis nigrican

A

B. Type 2 diabetes

is a disorder usually occurring around the age of 40, but it is now being detected in children and young adults as a result of obesity and sedentary lifestyles. Non-healing wounds are a hallmark sign of type 2 diabetes. This client weights 248.6 lbs and is short.

How well did you know this?
1
Not at all
2
3
4
5
Perfectly
81
Q

An adolescent client with type I diabetes mellitus is admitted to the emergency department for treatment of diabetic ketoacidosis. Which assessment findings should the nurse expect to note? “

a) sweating and tremors
b) hunger and hypertension
c) cold, clammy skin and irritability
d) fruity breath and decreasing level of consciousness

A

d) fruity breath and decreasing level of consciousness

“Hyperglycemia occurs with diabetic ketoacidosis. Signs of hyperglycemia include fruity breath and a decreasing level of consciousness. Hunger can be a sign of hypoglycemia or hyperglycemia, but hypertension is not a sign of diabetic ketoacidosis. Instead, hypotension occurs because of a decrease in blood volume related to the dehydrated state that occurs during diabetic ketoacidosis. Cold, clammy skin, irritability, sweating, and tremors are all signs of hypoglycemia.”

How well did you know this?
1
Not at all
2
3
4
5
Perfectly
82
Q

An external insulin pump is prescribed for a client with DM. The client asks the nurse about the functioning of the pump. The nurse bases the response on the information that the pump: “

a. Gives small continuous dose of regular insulin subcutaneously, and the client can self-administer a bolus with an additional dosage from the pump before each meal.
b. Is timed to release programmed doses of regular or NPH insulin into the bloodstream at specific intervals.
c. Is surgically attached to the pancreas and infuses regular insulin into the pancreas, which in turn releases the insulin into the bloodstream.
d. Continuously infuses small amounts of NPH insulin into the bloodstream while regularly monitoring blood glucose levels”

A

a. Gives small continuous dose of regular insulin subcutaneously, and the client can self-administer a bolus with an additional dosage from the pump before each meal.

An insulin pump provides a small continuous dose of regular insulin subcutaneously throughout the day and night, and the client can self-administer a bolus with additional dosage from the pump before each meal as needed. Regular insulin is used in an insulin pump. An external pump is not attached surgically to the pancreas.

How well did you know this?
1
Not at all
2
3
4
5
Perfectly
83
Q

Analyze the following diagnostic findings for your patient with type 2 diabetes. Which result will need further assessment?

A) BP 126/80

B) A1C 9%

C)FBG 130mg/dL

D) LDL cholesterol 100mg/dL

A

“B) A1C 9%

Rationale: Lowering hemoglobin A1C (to average of 7%) reduces microvascular and neuropathic complications. Tighter glycemic control(normal A1C < 6%) may further reduce complications but increases hypoglycemia risk.”

How well did you know this?
1
Not at all
2
3
4
5
Perfectly
84
Q

Blood sugar is well controlled when Hemoglobin A1C is… “

a. Below 7%
b. Between 12%-15%
c. Less than 180 mg/dL
d. Between 90 and 130 mg/dL”

A

a. Below 7%

A1c measures the percentage of hemoglobin that is glycated and determines average blood glucose during the 2 to 3 months prior to testing. Used as a diagnostic tool, A1C levels of 6.5% or higher on two tests indicate diabetes. A1C of 6% to 6.5% is considered prediabetes.”

How well did you know this?
1
Not at all
2
3
4
5
Perfectly
85
Q

Excessive thirst and volume of very dilute urine may be symptoms of: “

A. Urinary tract infection

B. Diabetes insipidus

C. Viral gastroenteritis

D.Hypoglycemia”

A

B. Diabetes insipidus

Diabetes insipidus is a condition in which the kidneys are unable to conserve water, often because there is insufficient antidiuretic hormone (ADH) or the kidneys are unable to respond to ADH. Although diabetes mellitus may present with similar symptoms, the disorders are different. Diabetes insipidus does not involve hyperglycemia.”

How well did you know this?
1
Not at all
2
3
4
5
Perfectly
86
Q

In educating a client with diabetes, what response would reveal need for further education? “

A. I should avoid tights

B. I should take good care of my toe nails

C. I should not go more than 3 days without washing my feet

D. I should avoid going barefoot and should wear clean socks

A

C. I should not go more than 3 days without washing my feet

“The recommended self-care routine is to wash feet on a daily basis without soaking and carefully cleaning.”

How well did you know this?
1
Not at all
2
3
4
5
Perfectly
87
Q

Of which of the following symptoms might an older woman with diabetes mellitus complain?

1) anorexia
2) pain intolerance
3) weight loss
4) perineal itching

A

4) perineal itching

Rationale: Older women might complain of perineal itching due to vaginal candidiasis.

How well did you know this?
1
Not at all
2
3
4
5
Perfectly
88
Q

One of the benefits of Glargine (Lantus) insulin is its ability to:

a. Release insulin rapidly throughout the day to help control basal glucose.
b. Release insulin evenly throughout the day and control basal glucose levels.
c. Simplify the dosing and better control blood glucose levels during the day.
d. Cause hypoglycemia with other manifestation of other adverse reactions.

A

B)Release insulin evenly throughout the day and control basal glucose levels

“Glargine (Lantus) insulin is designed to release insulin evenly throughout the day and control basal glucose levels

How well did you know this?
1
Not at all
2
3
4
5
Perfectly
89
Q

Prediabetes is associated with all of the following except: “

a. Increased risk of developing type 2 diabetes
b. Impaired glucose tolerance
c. Increased risk of heart disease and stroke
d. Increased risk of developing type 1 diabetes”

A

d. Increased risk of developing type 1 diabetes

Persons with elevated glucose levels that do not yet meet the criteria for diabetes are considered to have prediabetes and are at increased risk of developing type 2 diabetes. Weight loss and increasing physical activity can help people with prediabetes prevent or postpone the onset of type 2 diabetes.”

How well did you know this?
1
Not at all
2
3
4
5
Perfectly
90
Q

Risk factors for type 2 diabetes include all of the following except:

a. Advanced age
b. Obesity
c. Smoking
d. Physical inactivity”

A

c. Smoking

“Additional risk factors for type 2 diabetes are a family history of diabetes, impaired glucose metabolism, history of gestational diabetes, and race/ethnicity. African-Americans, Hispanics/Latinos, Asian Americans, Native Hawaiians, Pacific Islanders, and Native Americans are at greater risk of developing diabetes than whites.”

How well did you know this?
1
Not at all
2
3
4
5
Perfectly
91
Q

The client diagnosed with Type I diabetes is found lying unconscious on the floor of the bathroom. Which interventions should the nurse implement first?

A. Administer 50% dextrose IVP.

B. Notify the health-care provider.

C. Move the client to ICD.

D. Check the serum glucose level

A

A. Administer 50% dextrose IVP.

The nurse should assume the client is hypoglycemic and administer IVP dextrose, which will rouse the client immediately. If the collapse is the result of hyperglycemia, this additional dextrose will not further injure the client.

How well did you know this?
1
Not at all
2
3
4
5
Perfectly
92
Q

The client, an 18-year-old female, 5’4’’ tall, weighing 113 kg, comes to the clinic for a wound on her lower leg that has not healed for the last two (2) weeks. Which diseaseprocess would the nurse suspect that the client has developed? “

  1. Type 1 diabetes.
  2. Type 2 diabetes.
  3. Gestational diabetes.
  4. Acanthosis nigricans
A

2.Type 2 diabetes.

Type 2 diabetes is a disorder that usually occurs around the age of 40, but it is now being detected in children and young adultsas a result of obesity and sedentary life-styles. Wounds that do not heal are a hall-mark sign of Type 2 diabetes. This client weighs 248.6 pounds and is short”

How well did you know this?
1
Not at all
2
3
4
5
Perfectly
93
Q

The guidelines for Carbohydrate Counting as medical nutrition therapy for diabetes mellitus includes all of the following EXCEPT:

a. Flexibility in types and amounts of foods consumed
b. Unlimited intake of total fat, saturated fat and cholesterol
c. Including adequate servings of fruits, vegetables and the dairy group
d. Applicable to with either Type 1 or Type 2 diabetes mellitusb. Unlimited intake of total fat, saturated fat and cholesterol”

A

b. Unlimited intake of total fat, saturated fat and cholesterol

You want to be careful of how much you eat in any food group

How well did you know this?
1
Not at all
2
3
4
5
Perfectly
94
Q

The nurse administered 28 units of Humulin N, an intermediate-acting insulin, to a client diagnosed with Type 1 diabetes at 1600. Which action should the nurse implement? “

  1. Ensure the client eats the bedtime snack.
  2. Determine how much food the client ate at lunch.
  3. Perform a glucometer reading at 0700.
  4. Offer the client protein after administering insulin.
A
  1. Ensure the client eats the bedtime snack.

Humulin N peaks in 6-8 hours, making the client at risk for hypoglycemia around midnight, which is why the client should receive a bedtime snack. This snack will prevent nighttime hypoglycemia. (Correct)

How well did you know this?
1
Not at all
2
3
4
5
Perfectly
95
Q

The nurse assisting in the admission of a client with diabetic ketoacidosis will anticipate the physician ordering which of the following types of intravenous solution if the client cannot take any fluids orally? “

a. 0.45% normal saline solution
b. Lactated Ringer’s solution
c. 0.9 normal saline solution
d. 5% dextrose in water (D5W)”

A

a. 0.45% normal saline solution

normal saline solution Helps to hydrate patient and keep electrolyte levels balanced

How well did you know this?
1
Not at all
2
3
4
5
Perfectly
96
Q

The nurse caring for a 54-year-old patient hospitalized with diabetes mellitus would look for which of the following laboratory test results to obtain information on the patient’s past glucose control?

a. prealbumin level
b. urine ketone level
c. fasting glucose level
d. glycosylated hemoglobin level

A

d. glycosylated hemoglobin level

A glycosylated hemoglobin level detects the amount of glucose that is bound to red blood cells (RBCs). When circulating glucose levels are high, glucose attaches to the RBCs and remains there for the life of the blood cell, which is approximately 120 days. Thus the test can give an indication of glycemic control over approximately 2 to 3 months.

How well did you know this?
1
Not at all
2
3
4
5
Perfectly
97
Q

EKG (1,2)

A
How well did you know this?
1
Not at all
2
3
4
5
Perfectly
98
Q

EKG (3)

A

Another EKG interpretation :)

How well did you know this?
1
Not at all
2
3
4
5
Perfectly
99
Q

The nurse is caring for a client who has normal glucose levels at bedtime, hypoglycemia at 2am and hyperglycemia in the morning. What is this client likely experiencing? “

A. Dawn phenomenon

B. Somogyi effect

C. An insulin spike

D. Excessive corticosteroids”

A

B. Somogyi effect

The Somogyi effect is when blood sugar drops too low in the morning causing rebound hyperglycemia in the morning. The hypoglycemia at 2am is highly indicative. The Dawn phenomenon is similar but would not have the hypoglycemia at 2am.”

How well did you know this?
1
Not at all
2
3
4
5
Perfectly
100
Q

The nurse is caring for a client with long-term Type 2 diabetes and is assessing the feet. Which assessment data would warrant immediate intervention by the nurse? “

1) The client has crumbling toenails
2) The client has athlete’s feet
3) The client has a necrotic big toe
4) The client has thickened toenails.”

A

3)The client has a necrotic big toe

Crumbling toenails indicate tinea unguium, which is a fungus infection of the toenail. 2)Athlete’s foot is a fungal infection that is not life threatening. 3)A necrotic big toe indicates “dead” tissue. The client does not feel pain in the lower extremity and does not realize there has been an injury and therefore does not seek treatment. Increased blood glucose levels decrease oxygen supply that is needed to heal the wound and increase the risk for developing an infection. 4)Big, thick toenails are fungal infections and would not require immediate intervention by the nurse; 50% of the adult population has this.”]

How well did you know this?
1
Not at all
2
3
4
5
Perfectly
101
Q

The nurse is caring for a patient whose blood glucose level is 55mg/dL. What is the likely nursing response? “

A. Administer a glucagon injection

B. Give a small meal

C. Administer 10-15 g of a carbohydrate

D. Give a small snack of high protein food

A

C. Administer 10-15 g of a carbohydrate

The client has low hypoglycemia. This is generally treated with a small snack.”

How well did you know this?
1
Not at all
2
3
4
5
Perfectly
102
Q

The nurse is caring for a woman at 37 weeks gestation. The client was diagnosed with insulin-dependent diabetes mellitis (IDDM) at age 7. The client states, ““I am so thrilled that I will be breastfeeding my baby.”” Which of the following responses by the nurse is best? “

  1. You will probably need less insulin while you are breastfeeding.
  2. You will need to initially increase your insulin after the baby is born.
  3. You will be able to take an oral hypoglycemic instead of insulin after the baby is born.
  4. You will probably require the same dose of insulin that you are now taking.
A
  1. You will probably need less insulin while you are breastfeeding.
  2. breastfeeding has an antidiabetogenic effect, less insulin is needed. (correct) 2. insulin needs will decrease due to antidiabetogenic effect of breastfeeding and physiological changes during immediate postpartum period. 3. client has IDDM, insulin required. 4. during third trimester insulin requirements increase due to increased insulin resistance”
How well did you know this?
1
Not at all
2
3
4
5
Perfectly
103
Q

The nurse is discharging a client diagnosed with diabetes insipidus. Which statementmade by the client warrants further intervention? “

  1. “I will keep a list of my medications in my wallet and wear a Medi bracelet.”
  2. “I should take my medication in the morning and leave it refrigerated at home.”
  3. “I should weigh myself every morning and record any weight gain.”
  4. “If I develop a tightness in my chest, I will call my health-care provider.”
A

2.”I should take my medication in the morning and leave it refrigerated at home.”

I should take my medication in the morning and leave it refrigerated at home.”“1.The client should keep a list of medication being taken and wear a Medic Alert bracelet. 2. Medication taken for DI is usually every 8-12 hours, depending on the client. Theclient should keep the medication close at hand. 3.The client is at risk for fluid shifts. Weighing every morning allows the client to follow thefluid shifts. Weight gain could indicate too much medication. 4.Tightness in the chest could be an indicator that the medication is not being tolerated; if this occurs the client should call the health-care provider”

How well did you know this?
1
Not at all
2
3
4
5
Perfectly
104
Q

The nurse is discussing the importance of exercising to a client diagnosed with Type 2diabetes whose diabetes is well controlled with diet and exercise. Which informationshould the nurse include in the teaching about diabetes? “

  1. Eat a simple carbohydrate snack before exercising.
  2. Carry peanut butter crackers when exercising.
  3. Encourage the client to walk 20 minutes three (3) times a week.
  4. Perform warmup and cooldown exercises
A

4.Perform warmup and cooldown exercises

All clients who exercise should perform warmup and cooldown exercises to helpprevent muscle strain and injury”

How well did you know this?
1
Not at all
2
3
4
5
Perfectly
105
Q

The nurse is educating a pregnant client who has gestational diabetes. Which of the following statements should the nurse make to the client? Select all that apply. “

a. Cakes, candies, cookies, and regular soft drinks should be avoided.
b. Gestational diabetes increases the risk that the mother will develop diabetes later in life.
c. Gestational diabetes usually resolves after the baby is born.
d. Insulin injections may be necessary.
e. The baby will likely be born with diabetes f. The mother should strive to gain no more weight during the pregnancy.

A

a. Cakes, candies, cookies, and regular soft drinks should be avoided.
b. Gestational diabetes increases the risk that the mother will develop diabetes later in life.
c. Gestational diabetes usually resolves after the baby is born.
d. Insulin injections may be necessary.

Gestational diabetes can occur between the 16th and 28th week of pregnancy. If not responsive to diet and exercise, insulin injections may be necessary. Concentrated sugars should be avoided. Weight gain should continue, but not in excessive amounts. Usually, gestational diabetes disappears after the infant is born. However, diabetes can develop 5 to 10 years after the pregnancy”

How well did you know this?
1
Not at all
2
3
4
5
Perfectly
106
Q

The nurse is teaching a community class to peole with Type 2 diabetes mellitus. Which explanation would explain the development of Type 2 diabetes?

  1. The islet cells in the pancreas stop producing insulin.
  2. The client eats too many foods that are high in sugar.
  3. The pituitary gland does not produce vasopression.
  4. The cells become resistant to the circulating insulin.
A
  1. The cells become resistant to the circulating insulin.

Normally insulin binds to special receptor sites on the cells and initiates a series of reactions involved in metabolism. In Type 2 diabetes these reactions are diminished primarily as a result of obesity and aging.”

How well did you know this?
1
Not at all
2
3
4
5
Perfectly
107
Q

The nurse is working with an overweight client who has a high-stress job and smokes. This client has just received a diagnosis of Type II Diabetes and has just been started on an oral hypoglycemic agent. Which of the following goals for the client which if met, would be most likely to lead to an improvement in insulin efficiency to the point the client would no longer require oral hypoglycemic agents? “

a. Comply with medication regimen 100% for 6 months
b. Quit the use of any tobacco products by the end of three months
c. Lose a pound a week until weight is in normal range for height and exercise 30 minutes daily
d. Practice relaxation techniques for at least five minutes five times a day for at least five months”

A

c. Lose a pound a week until weight is in normal range for height and exercise 30 minutes daily

When type II diabetics lose weight through diet and exercise they sometimes have an improvement in insulin efficiency sufficient to the degree they no longer require oral hypoglycemic agents.

108
Q

What insulin type can be given by IV? Select all that apply: “

A. Glipizide (Glucotrol)

B. Lispro (Humalog)

C. NPH insulin

D. Glargine (Lantus)

E. Regular insulin

A

E) Regular insulin

The only insulin that can be given by IV is regular insulin.

109
Q

What will the nurse teach the client with diabetes regarding exercise in his or her treatment program?

  1. During exercise the body will use carbohydrates for energy production, which in turn will decrease the need for insulin.
  2. With an increase in activity, the body will use more carbohydrates; therefore more insulin will be required.
  3. The increase in activity results in an increase in the use of insulin; therefore the client should decrease his or her carbohydrate intake.
  4. Exercise will improve pancreatic circulation and stimulate the islets of Langerhans to increase the production of intrinsic insulin.
A
  1. During exercise the body will use carbohydrates for energy production, which in turn will decrease the need for insulin.

During exercise the body will use carbohydrates for energy production, which in turn will decrease the need for insuli”Rationale: As carbohydrates are used for energy, insulin needs decrease. Therefore during exercise, carbohydrate intake should be increased to cover the increased energy requirements. The beneficial effects of regular exercise may result in a decreased need for diabetic medications in order to reach target blood glucose levels. Furthermore, it may help to reduce triglycerides, LDL cholesterol levels, increase HDLs, reduce blood pressure, and improve circulation.”

110
Q

When an older adult is admitted to the hospital with a diagnosis of diabetes mellitus and complaints of rapid-onset weight loss, elevated blood glucose levels, and polyphagia, the gerontology nurse should anticipate which of the following secondary medical diagnoses? “

  1. Impaired glucose tolerance
  2. Gestational diabetes mellitus
  3. Pituitary tumor
  4. Pancreatic tumor
A
  1. Pancreatic tumor

Rationale: The onset of hyperglycemia in the older adult can occur more slowly. When the older adult reports rapid-onset weight loss, elevated blood glucose levels, and polyphagia, the healthcare provider should consider pancreatic tumor.”

111
Q

When assessing the patient experiencing the onset of symptoms of type 1 diabetes, which question should the nurse ask?

a. ““Have you lost any weight lately?””
b. ““Do you crave fluids containing sugar?””
c. ““How long have you felt anorexic?””
d. ““Is your urine unusually dark-colored?””

A

a. ““Have you lost any weight lately?””

Weight loss occurs because the body is no longer able to absorb glucose and starts to break down protein and fat for energy.

b. The patient is thirsty but does not necessarily crave sugar- containing fluids.
c. Increased appetite is a classic symptom of type 1 diabetes.
d. With the classic symptom of polyuria, urine will be very dilute.”

112
Q
A
113
Q

When taking a health history, the nurse screens for manifestations suggestive of diabetes type I. Which of the following manifestations are considered the primary manifestations of diabetes type I and would be most suggestive of diabetes type I and require follow-up investigation? “

a. Excessive intake of calories, rapid weight gain, and difficulty losing weight
b. Poor circulation, wound healing, and leg ulcers,
c. Lack of energy, weight gain, and depression
d. An increase in three areas: thirst, intake of fluids, and hunger

A

d. An increase in three areas: thirst, intake of fluids, and hunger

“The primary manifestations of diabetes type I are polyuria (increased urine output), polydipsia (increased thirst), polyphagia (increased hunger). Excessive calorie intake, weight gain, and difficulty losing weight are common risk factors for type 2 diabetes. Poor circulation, wound healing and leg ulcers are signs of chronic diabetes. Lack of energy, weight gain and depression are not necessarily indicative of any type of diabetes.”

114
Q

which are symptoms of hypoglycemia?

A. irritability,

B. drowsiness

c. Abdominal pain

D. nausea and vomiting

A

A. Irritability:

signs of hypoglycemia include irritability, shaky feeling, hunger, headache, dizziness. Other symptoms are hyperglycemia.

115
Q

Which electrolyte replacement should the nurse anticipate being ordered by thehealth-care provider in the client diagnosed with DKA who has just been admitted tothe ICD?

  1. Glucose.
  2. Potassium.
  3. Calcium.
  4. Sodium
A

2.Potassium.

The client in DKA loses potassium from increased urinary output, acidosis, cata-bolic state, and vomiting. Replacement isessential for preventing cardiac dysrhyth-mias secondary to hypokalemia.

116
Q

Which of the following factors are risks for the development of diabetes mellitus? (Select all that apply.)

a) Age over 45 years
b) Overweight with a waist/hip ratio >1
c) Having a consistent HDL level above 40 mg/dl
d) Maintaining a sedentary lifestyle

A

a) Age over 45 years
b) Overweight with a waist/hip ratio >1
d) Maintaining a sedentary lifestyle

Aging results in reduced ability of beta cells to respond with insulin effectively. Overweight with waist/hip ratio increase is part of the metabolic syndrome of DM II. There is an increase in atherosclerosis with DM due to the metabolic syndrome and sedentary lifestyle.

117
Q

Which of the following is accurate pertaining to physical exercise and type 1 diabetes mellitus? “

  1. Physical exercise can slow the progression of diabetes mellitus.
  2. Strenuous exercise is beneficial when the blood glucose is high.
  3. Patients who take insulin and engage in strenuous physical exercise might experience hyperglycemia.
  4. Adjusting insulin regimen allows for safe participation in all forms of exercise.”
A

1) physical exercise can slow the progression of diabetes mellitus

Rationale: Physical exercise slows the progression of diabetes mellitus, because exercise has beneficial effects on carbohydrate metabolism and insulin sensitivity. Strenuous exercise can cause retinal damage, and can cause hypoglycemia. Insulin and foods both must be adjusted to allow safe participation in exercise.

118
Q

Which of the following persons would most likely be diagnosed with diabetes mellitus? A 44-year-old.. “

A. Caucasian Woman

B. Asian Woman

C. African-American woman

D. Hispanic Male

A

C. African-American woman

African-American woman Rationale: Age-specific prevalence of diagnosed diabetes mellitus (DM) is higher for African-Americans and Hispanics than for Caucasians. Among those younger than 75, black women had the highest incidence.”

119
Q

Which statement by the patient with type 2 diabetes is accurate. “

a. “I am supposed to have a meal or snak if I drink alcohol”
b. “I am not allowed to eat any sweets because of my diabetes.”
c. I do not need to watch what I eat because my diabetes is not the bad kind.”
d. The amunt of fat in my diet is not important; it is just the carbohydrates that raise my blood sugar.

A

a. “I am supposed to have a meal or snak if I drink alcohol”

Alcohol should be consumed with food to reduce the risk of hypoglycemia.”

120
Q

A 68-year-old patient has been in the intensive care unit for 4 days and has a nursing diagnosis of disturbed sensory perception related to sleep deprivation. Which action should the nurse include in the plan of care?

a. Administer prescribed sedatives or opioids at bedtime to promote sleep.
b. Cluster nursing activities so that the patient has uninterrupted rest periods.
c. Silence the alarms on the cardiac monitors to allow 30- to 40-minute naps.
d. Eliminate assessments between 0100 and 0600 to allow uninterrupted sleep.

A

b. Cluster nursing activities so that the patient has uninterrupted rest periods.

Clustering nursing activities and providing uninterrupted rest periods will minimize sleep-cycle disruption. Sedative and opioid medications tend to decrease the amount of rapid eye movement (REM) sleep and can contribute to sleep disturbance and disturbed sensory perception. Silencing the alarms on the cardiac monitors would be unsafe in a critically ill patient, as would discontinuing assessments during the night.

121
Q

Which hemodynamic parameter is most appropriate for the nurse to monitor to determine the effectiveness of medications given to a patient to reduce left ventricular afterload?

a. Mean arterial pressure (MAP)
b. Systemic vascular resistance (SVR)
c. Pulmonary vascular resistance (PVR)
d. Pulmonary artery wedge pressure (PAWP)

A

b. Systemic vascular resistance (SVR)

Systemic vascular resistance reflects the resistance to ventricular ejection, or afterload. The other parameters will be monitored, but do not reflect afterload as directly

122
Q

While family members are visiting, a patient has a respiratory arrest and is being resuscitated. Which action by the nurse is best?

a. Tell the family members that watching the resuscitation will be very stressful.
b. Ask family members if they wish to remain in the room during the resuscitation.
c. Take the family members quickly out of the patient room and remain with them.
d. Assign a staff member to wait with family members just outside the patient room.

A

b. Ask family members if they wish to remain in the room during the resuscitation.

Research indicates that family members want the option of remaining in the room during procedures such as cardiopulmonary resuscitation (CPR) and that this decreases anxiety and facilitates grieving. The other options may be appropriate if the family decides not to remain with the patient.

123
Q

Following surgery for an abdominal aortic aneurysm, a patient’s central venous pressure (CVP) monitor indicates low pressures. Which action is a priority for the nurse to take?

a. Administer IV diuretic medications.
b. Increase the IV fluid infusion per protocol.
c. Document the CVP and continue to monitor.
d. Elevate the head of the patient’s bed to 45 degrees.

A

b. Increase the IV fluid infusion per protocol.

A low CVP indicates hypovolemia and a need for an increase in the infusion rate. Diuretic administration will contribute to hypovolemia and elevation of the head may decrease cerebral perfusion. Documentation and continued monitoring is an inadequate response to the low CVP.

124
Q

When caring for a patient with pulmonary hypertension, which parameter is most appropriate for the nurse to monitor to evaluate the effectiveness of the treatment?

a. Central venous pressure (CVP)
b. Systemic vascular resistance (SVR)
c. Pulmonary vascular resistance (PVR)
d. Pulmonary artery wedge pressure (PAWP)

A

c. Pulmonary vascular resistance (PVR)

PVR is a major contributor to pulmonary hypertension, and a decrease would indicate that pulmonary hypertension was improving. The other parameters also may be monitored but do not directly assess for pulmonary hypertension.

125
Q

The intensive care unit (ICU) nurse educator will determine that teaching about arterial pressure monitoring for a new staff nurse has been effective when the nurse

a. balances and calibrates the monitoring equipment every 2 hours.
b. positions the zero-reference stopcock line level with the phlebostatic axis.
c. ensures that the patient is supine with the head of the bed flat for all readings.
d. rechecks the location of the phlebostatic axis when changing the patient’s position.

A

b. positions the zero-reference stopcock line level with the phlebostatic axis.

For accurate measurement of pressures, the zero-reference level should be at the phlebostatic axis. There is no need to rebalance and recalibrate monitoring equipment hourly. Accurate hemodynamic readings are possible with the patient’s head raised to 45 degrees or in the prone position. The anatomic position of the phlebostatic axis does not change when patients are repositioned.

126
Q

When monitoring for the effectiveness of treatment for a patient with a large anterior wall myocardial infarction, the most important information for the nurse to obtain is

a. central venous pressure (CVP).
b. systemic vascular resistance (SVR).
c. pulmonary vascular resistance (PVR).
d. pulmonary artery wedge pressure (PAWP).

A

d. pulmonary artery wedge pressure (PAWP).

PAWP reflects left ventricular end diastolic pressure (or left ventricular preload) and is a sensitive indicator of cardiac function. Because the patient is high risk for left ventricular failure, the PAWP must be monitored. An increase will indicate left ventricular failure. The other values would also provide useful information, but the most definitive measurement of changes in cardiac function is the PAWP.

127
Q

Which action is a priority for the nurse to take when the low pressure alarm sounds for a patient who has an arterial line in the left radial artery?

a. Fast flush the arterial line.
b. Check the left hand for pallor.
c. Assess for cardiac dysrhythmias.
d. Rezero the monitoring equipment.

A

c. Assess for cardiac dysrhythmias.

The low pressure alarm indicates a drop in the patient’s blood pressure, which may be caused by cardiac dysrhythmias. There is no indication to rezero the equipment. Pallor of the left hand would be caused by occlusion of the radial artery by the arterial catheter, not by low pressure. There is no indication of a need for flushing the line.

128
Q

Which action will the nurse need to do when preparing to assist with the insertion of a pulmonary artery catheter?

a. Determine if the cardiac troponin level is elevated.
b. Auscultate heart and breath sounds during insertion.
c. Place the patient on NPO status before the procedure.
d. Attach cardiac monitoring leads before the procedure.

A

d. Attach cardiac monitoring leads before the procedure.

Dysrhythmias can occur as the catheter is floated through the right atrium and ventricle, and it is important for the nurse to monitor for these during insertion. Pulmonary artery catheter insertion does not require anesthesia, and the patient will not need to be NPO. Changes in cardiac troponin or heart and breath sounds are not expected during pulmonary artery catheter insertion.

129
Q

When assisting with the placement of a pulmonary artery (PA) catheter, the nurse notes that the catheter is correctly placed when the monitor shows a

a. typical PA pressure waveform.
b. tracing of the systemic arterial pressure.
c. tracing of the systemic vascular resistance.
d. typical PA wedge pressure (PAWP) tracing.

A

d. typical PA wedge pressure (PAWP) tracing.

The purpose of a PA line is to measure PAWP, so the catheter is floated through the pulmonary artery until the dilated balloon wedges in a distal branch of the pulmonary artery, and the PAWP readings are available. After insertion, the balloon is deflated and the PA waveform will be observed. Systemic arterial pressures are obtained using an arterial line and the systemic vascular resistance is a calculated value, not a waveform.

130
Q

Which assessment finding obtained by the nurse when caring for a patient with a right radial arterial line indicates a need for the nurse to take immediate action?

a. The right hand is cooler than the left hand.
b. The mean arterial pressure (MAP) is 77 mm Hg.
c. The system is delivering 3 mL of flush solution per hour.
d. The flush bag and tubing were last changed 3 days previously.

A

a. The right hand is cooler than the left hand.

The change in temperature of the left hand suggests that blood flow to the left hand is impaired. The flush system needs to be changed every 96 hours. A mean arterial pressure (MAP) of 75 mm Hg is normal. Flush systems for hemodynamic monitoring are set up to deliver 3 to 6 mL/hour of flush solution

131
Q

The central venous oxygen saturation (ScvO2) is decreasing in a patient who has severe pancreatitis. To determine the possible cause of the decreased ScvO2, the nurse assesses the patient’s

a. lipase.
b. temperature.
c. urinary output.
d. body mass index.

A

ANS: B: Temperature

Elevated temperature increases metabolic demands and oxygen use by tissues, resulting in a drop in oxygen saturation of central venous blood. Information about the patient’s body mass index, urinary output, and lipase will not help in determining the cause of the patient’s drop in ScvO2.

132
Q

An intraaortic balloon pump (IABP) is being used for a patient who is in cardiogenic shock. Which assessment data indicate to the nurse that the goals of treatment with the IABP are being met?

a. Urine output of 25 mL/hr
b. Heart rate of 110 beats/minute
c. Cardiac output (CO) of 5 L/min
d. Stroke volume (SV) of 40 mL/beat

A

c. Cardiac output (CO) of 5 L/min

A CO of 5 L/min is normal and indicates that the IABP has been successful in treating the shock. The low SV signifies continued cardiogenic shock. The tachycardia and low urine output also suggest continued cardiogenic shock.

133
Q

The nurse is caring for a patient who has an intraaortic balloon pump in place. Which action should be included in the plan of care?

a. Position the patient supine at all times.
b. Avoid the use of anticoagulant medications.
c. Measure the patient’s urinary output every hour.
d. Provide passive range of motion for all extremities.

A

c. Measure the patient’s urinary output every hour.

Monitoring urine output will help determine whether the patient’s cardiac output has improved and also help monitor for balloon displacement. The head of the bed can be elevated up to 30 degrees. Heparin is used to prevent thrombus formation. Limited movement is allowed for the extremity with the balloon insertion site to prevent displacement of the balloon.

134
Q

While waiting for cardiac transplantation, a patient with severe cardiomyopathy has a ventricular assist device (VAD) implanted. When planning care for this patient, the nurse should anticipate

a. giving immunosuppressive medications.
b. preparing the patient for a permanent VAD.
c. teaching the patient the reason for complete bed rest.
d. monitoring the surgical incision for signs of infection.

A

d. monitoring the surgical incision for signs of infection.

The insertion site for the VAD provides a source for transmission of infection to the circulatory system and requires frequent monitoring. Patient’s with VADs are able to have some mobility and may not be on bed rest. The VAD is a bridge to transplantation, not a permanent device. Immunosuppression is not necessary for nonbiologic devices like the VAD.

135
Q

To verify the correct placement of an oral endotracheal tube (ET) after insertion, the best initial action by the nurse is to

a. auscultate for the presence of bilateral breath sounds.
b. obtain a portable chest x-ray to check tube placement.
c. observe the chest for symmetric chest movement with ventilation.
d. use an end-tidal CO2 monitor to check for placement in the trachea.

A

d. use an end-tidal CO2 monitor to check for placement in the trachea.

End-tidal CO2 monitors are currently recommended for rapid verification of ET placement. Auscultation for bilateral breath sounds and checking chest expansion are also used, but they are not as accurate as end-tidal CO2 monitoring. A chest x-ray confirms the placement but is done after the tube is secured.

136
Q

The nurse notes thick, white secretions in the endotracheal tube (ET) of a patient who is receiving mechanical ventilation. Which intervention will be most effective in addressing this problem?

a. Increase suctioning to every hour.
b. Reposition the patient every 1 to 2 hours.
c. Add additional water to the patient’s enteral feedings.
d. Instill 5 mL of sterile saline into the ET before suctioning.

A

c. Add additional water to the patient’s enteral feedings.

Because the patient’s secretions are thick, better hydration is indicated. Suctioning every hour without any specific evidence for the need will increase the incidence of mucosal trauma and would not address the etiology of the ineffective airway clearance. Instillation of saline does not liquefy secretions and may decrease the SpO2. Repositioning the patient is appropriate but will not decrease the thickness of secretions.

137
Q

Four hours after mechanical ventilation is initiated for a patient with chronic obstructive pulmonary disease (COPD), the patient’s arterial blood gas (ABG) results include a pH of 7.51, PaO2 of 82 mm Hg, PaCO2 of 26 mm Hg, and HCO3- of 23 mEq/L (23 mmol/L). The nurse will anticipate the need to

a. increase the FIO2.
b. increase the tidal volume.
c. increase the respiratory rate.
d. decrease the respiratory rate.

A

d. decrease the respiratory rate.

The patient’s PaCO2 and pH indicate respiratory alkalosis caused by too high a respiratory rate. The PaO2 is appropriate for a patient with COPD and increasing the respiratory rate and tidal volume would further lower the PaCO2.

138
Q

A patient with respiratory failure has arterial pressure-based cardiac output (APCO) monitoring and is receiving mechanical ventilation with peak end-expiratory pressure (PEEP) of 12 cm H2O. Which information indicates that a change in the ventilator settings may be required?

a. The arterial pressure is 90/46.
b. The heart rate is 58 beats/minute.
c. The stroke volume is increased.
d. The stroke volume variation is 12%.

A

a. The arterial pressure is 90/46.

The hypotension suggests that the high intrathoracic pressure caused by the PEEP may be decreasing venous return and (potentially) cardiac output. The other assessment data would not be a direct result of PEEP and mechanical ventilation.

139
Q

The nurse is caring for a patient receiving a continuous norepinephrine (Levophed) IV infusion. Which patient assessment finding indicates that the infusion rate may need to be adjusted?

a. Heart rate is 58 beats/minute.
b. Mean arterial pressure (MAP) is 56 mm Hg.
c. Systemic vascular resistance (SVR) is elevated.
d. Pulmonary artery wedge pressure (PAWP) is low.

A

c. Systemic vascular resistance (SVR) is elevated.

Vasoconstrictors such as norepinephrine (Levophed) will increase SVR, and this will increase the work of the heart and decrease peripheral perfusion. The infusion rate may need to be decreased. Bradycardia, hypotension (MAP of 56 mm Hg), and low PAWP are not associated with norepinephrine infusion.

140
Q

When caring for the patient with a pulmonary artery (PA) pressure catheter, the nurse observes that the PA waveform indicates that the catheter is in the wedged position. Which action should the nurse take next?

a. Zero balance the transducer.
b. Activate the fast flush system.
c. Notify the health care provider.
d. Deflate and reinflate the PA balloon.

A

d. Deflate and reinflate the PA balloon.

When the catheter is in the wedge position, blood flow past the catheter is obstructed, placing the patient at risk for pulmonary infarction. A health care provider or advanced practice nurse should be called to reposition the catheter. The other actions will not correct the wedging of the PA catheter.

141
Q

When evaluating a patient with a central venous catheter, the nurse observes that the insertion site is red and tender to touch and the patient’s temperature is 101.8° F. What should the nurse plan to do next?

a. Give analgesics and antibiotics as ordered.
b. Discontinue the catheter and culture the tip.
c. Change the flush system and monitor the site.
d. Check the site more frequently for any swelling.

A

b. Discontinue the catheter and culture the tip.

The information indicates that the patient has a local and systemic infection caused by the catheter, and the catheter should be discontinued. Changing the flush system, giving analgesics, and continued monitoring will not help prevent or treat the infection. Administration of antibiotics is appropriate, but the line should still be discontinued to avoid further complications such as endocarditis.

142
Q

An 81-year-old patient who has been in the intensive care unit (ICU) for a week is now stable and transfer to the progressive care unit is planned. On rounds, the nurse notices that the patient has new onset confusion. The nurse will plan to

a. give PRN lorazepam (Ativan) and cancel the transfer.
b. inform the receiving nurse and then transfer the patient.
c. notify the health care provider and postpone the transfer.
d. obtain an order for restraints as needed and transfer the patient.

A

b. inform the receiving nurse and then transfer the patient.

The patient’s history and symptoms most likely indicate delirium associated with the sleep deprivation and sensory overload in the ICU environment. Informing the receiving nurse and transferring the patient is appropriate. Postponing the transfer is likely to prolong the delirium. Benzodiazepines and restraints contribute to delirium and agitation.

143
Q

The family members of a patient who has just been admitted to the intensive care unit (ICU) with multiple traumatic injuries have just arrived in the ICU waiting room. Which action should the nurse take next?

a. Explain ICU visitation policies and encourage family visits.
b. Immediately take the family members to the patient’s bedside.
c. Describe the patient’s injuries and the care that is being provided.
d. Invite the family to participate in a multidisciplinary care conference.

A

c. Describe the patient’s injuries and the care that is being provided.

Lack of information is a major source of anxiety for family members and should be addressed first. Family members should be prepared for the patient’s appearance and the ICU environment before visiting the patient for the first time. ICU visiting should be individualized to each patient and family rather than being dictated by rigid visitation policies. Inviting the family to participate in a multidisciplinary conference is appropriate but should not be the initial action by the nurse.

144
Q

The nurse responds to a ventilator alarm and finds the patient lying in bed holding the endotracheal tube (ET). Which action should the nurse take next?

a. Activate the rapid response team.
b. Provide reassurance to the patient.
c. Call the health care provider to reinsert the tube.
d. Manually ventilate the patient with 100% oxygen.

A

d. Manually ventilate the patient with 100% oxygen.

The nurse should ensure maximal patient oxygenation by manually ventilating with a bag-valve-mask system. Offering reassurance to the patient, notifying the health care provider about the need to reinsert the tube, and activating the rapid response team are also appropriate after the nurse has stabilized the patient’s oxygenation

145
Q

The nurse educator is evaluating the care that a new registered nurse (RN) provides to a patient receiving mechanical ventilation. Which action by the new RN indicates the need for more education?

a. The RN increases the FIO2 to 100% before suctioning.
b. The RN secures a bite block in place using adhesive tape.
c. The RN asks for assistance to reposition the endotracheal tube.
d. The RN positions the patient with the head of bed at 10 degrees.

A

d. The RN positions the patient with the head of bed at 10 degrees.

The head of the patient’s bed should be positioned at 30 to 45 degrees to prevent ventilator-associated pneumonia. The other actions by the new RN are appropriate.

146
Q

When assessing a patient who spilled hot oil on the right leg and foot, the nurse notes that the skin is dry, pale, hard skin. The patient states that the burn is not painful. What term would the nurse use to document the burn depth?

a. First-degree skin destruction
b. Full-thickness skin destruction
c. Deep partial-thickness skin destruction
d. Superficial partial-thickness skin destruction

A

b. Full-thickness skin destruction

With full-thickness skin destruction, the appearance is pale and dry or leathery and the area is painless because of the associated nerve destruction. Erythema, swelling, and blisters point to a deep partial-thickness burn. With superficial partial-thickness burns, the area is red, but no blisters are present. First-degree burns exhibit erythema, blanching, and pain

147
Q

A patient is admitted to the burn unit with burns to the head, face, and hands. Initially, wheezes are heard, but an hour later, the lung sounds are decreased and no wheezes are audible. What is the best action for the nurse to take?

a. Encourage the patient to cough and auscultate the lungs again.
b. Notify the health care provider and prepare for endotracheal intubation.
c. Document the results and continue to monitor the patient’s respiratory rate.
d. Reposition the patient in high-Fowler’s position and reassess breath sounds.

A

b. Notify the health care provider and prepare for endotracheal intubation.

The patient’s history and clinical manifestations suggest airway edema and the health care provider should be notified immediately, so that intubation can be done rapidly. Placing the patient in a more upright position or having the patient cough will not address the problem of airway edema. Continuing to monitor is inappropriate because immediate action should occur.

148
Q

During the emergent phase of burn care, which assessment will be most useful in determining whether the patient is receiving adequate fluid infusion?

a. Check skin turgor.
b. Monitor daily weight.
c. Assess mucous membranes.
d. Measure hourly urine output.

A

d. Measure hourly urine output.

When fluid intake is adequate, the urine output will be at least 0.5 to 1 mL/kg/hour. The patient’s weight is not useful in this situation because of the effects of third spacing and evaporative fluid loss. Mucous membrane assessment and skin turgor also may be used, but they are not as adequate in determining that fluid infusions are maintaining adequate perfusion.

149
Q

A patient has just been admitted with a 40% total body surface area (TBSA) burn injury. To maintain adequate nutrition, the nurse should plan to take which action?

a. Insert a feeding tube and initiate enteral feedings.
b. Infuse total parenteral nutrition via a central catheter.
c. Encourage an oral intake of at least 5000 kcal per day.
d. Administer multiple vitamins and minerals in the IV solution.

A

a. Insert a feeding tube and initiate enteral feedings.

Enteral feedings can usually be initiated during the emergent phase at low rates and increased over 24 to 48 hours to the goal rate. During the emergent phase, the patient will be unable to eat enough calories to meet nutritional needs and may have a paralytic ileus that prevents adequate nutrient absorption. Vitamins and minerals may be administered during the emergent phase, but these will not assist in meeting the patient’s caloric needs. Parenteral nutrition increases the infection risk, does not help preserve gastrointestinal function, and is not routinely used in burn patients.

150
Q

While the patient’s full-thickness burn wounds to the face are exposed, what is the best nursing action to prevent cross contamination?

a. Use sterile gloves when removing old dressings.
b. Wear gowns, caps, masks, and gloves during all care of the patient.
c. Administer IV antibiotics to prevent bacterial colonization of wounds.
d. Turn the room temperature up to at least 70° F (20° C) during dressing changes.

A

b. Wear gowns, caps, masks, and gloves during all care of the patient.

Use of gowns, caps, masks, and gloves during all patient care will decrease the possibility of wound contamination for a patient whose burns are not covered. When removing contaminated dressings and washing the dirty wound, use nonsterile, disposable gloves. The room temperature should be kept at approximately 85° F for patients with open burn wounds to prevent shivering. Systemic antibiotics are not well absorbed into deep burns because of the lack of circulation.

151
Q

A nurse is caring for a patient who has burns of the ears, head, neck, and right arm and hand. The nurse should place the patient in which position?

a. Place the right arm and hand flexed in a position of comfort.
b. Elevate the right arm and hand on pillows and extend the fingers.
c. Assist the patient to a supine position with a small pillow under the head.
d. Position the patient in a side-lying position with rolled towel under the neck.

A

b. Elevate the right arm and hand on pillows and extend the fingers.

The right hand and arm should be elevated to reduce swelling and the fingers extended to avoid flexion contractures (even though this position may not be comfortable for the patient). The patient with burns of the ears should not use a pillow for the head because this will put pressure on the ears, and the pillow may stick to the ears. Patients with neck burns should not use a pillow because the head should be maintained in an extended position in order to avoid contractures.

152
Q

A patient with circumferential burns of both legs develops a decrease in dorsalis pedis pulse strength and numbness in the toes. Which action should the nurse take?

a. Notify the health care provider.
b. Monitor the pulses every 2 hours.
c. Elevate both legs above heart level with pillows.
d. Encourage the patient to flex and extend the toes on both feet.

A

a. Notify the health care provider.

The decrease in pulse in a patient with circumferential burns indicates decreased circulation to the legs and the need for an escharotomy. Monitoring the pulses is not an adequate response to the decrease in circulation. Elevating the legs or increasing toe movement will not improve the patient’s circulation.

153
Q

Esomeprazole (Nexium) is prescribed for a patient who incurred extensive burn injuries 5 days ago. Which nursing assessment would best evaluate the effectiveness of the medication?

a. Bowel sounds
b. Stool frequency
c. Abdominal distention
d. Stools for occult blood

A

d. Stools for occult blood

H2 blockers and proton pump inhibitors are given to prevent Curling’s ulcer in the patient who has suffered burn injuries. Proton pump inhibitors usually do not affect bowel sounds, stool frequency, or appetite.

154
Q

The nurse is reviewing the medication administration record (MAR) on a patient with partial-thickness burns. Which medication is best for the nurse to administer before scheduled wound debridement?

a. Ketorolac (Toradol)
b. Lorazepam (Ativan)
c. Gabapentin (Neurontin)
d. Hydromorphone (Dilaudid)

A

d. Hydromorphone (Dilaudid)

Opioid pain medications are the best choice for pain control. The other medications are used as adjuvants to enhance the effects of opioids.

155
Q

A young adult patient who is in the rehabilitation phase after having deep partial-thickness face and neck burns has a nursing diagnosis of disturbed body image. Which statement by the patient indicates that the problem is resolving?

a. “I’m glad the scars are only temporary.”
b. “I will avoid using a pillow, so my neck will be OK.”
c. “I bet my boyfriend won’t even want to look at me anymore.”
d. “Do you think dark beige makeup foundation would cover this scar on my cheek?”

A

d. “Do you think dark beige makeup foundation would cover this scar on my cheek?”

The willingness to use strategies to enhance appearance is an indication that the disturbed body image is resolving. Expressing feelings about the scars indicates a willingness to discuss appearance, but not resolution of the problem. Because deep partial-thickness burns leave permanent scars, a statement that the scars are temporary indicates denial rather than resolution of the problem. Avoiding using a pillow will help prevent contractures, but it does not address the problem of disturbed body image

156
Q

The nurse caring for a patient admitted with burns over 30% of the body surface assesses that urine output has dramatically increased. Which action by the nurse would best ensure adequate kidney function?

a. Continue to monitor the urine output.
b. Monitor for increased white blood cells (WBCs).
c. Assess that blisters and edema have subsided.
d. Prepare the patient for discharge from the burn unit.

A

a. Continue to monitor the urine output.

The patient’s urine output indicates that the patient is entering the acute phase of the burn injury and moving on from the emergent stage. At the end of the emergent phase, capillary permeability normalizes and the patient begins to diurese large amounts of urine with a low specific gravity. Although this may occur at about 48 hours, it may be longer in some patients. Blisters and edema begin to resolve, but this process requires more time. White blood cells may increase or decrease, based on the patient’s immune status and any infectious processes. The WBC count does not indicate kidney function. The patient will likely remain in the burn unit during the acute stage of burn injury

157
Q

A patient with burns covering 40% total body surface area (TBSA) is in the acute phase of burn treatment. Which snack would be best for the nurse to offer to this patient?

a. Bananas
b. Orange gelatin
c. Vanilla milkshake
d. Whole grain bagel

A

c. Vanilla milkshake

A patient with a burn injury needs high protein and calorie food intake, and the milkshake is the highest in these nutrients. The other choices are not as nutrient-dense as the milkshake. Gelatin is likely high in sugar. The bagel is a good carbohydrate choice, but low in protein. Bananas are a good source of potassium, but are not high in protein and calories.

158
Q

A patient has just arrived in the emergency department after an electrical burn from exposure to a high-voltage current. What is the priority nursing assessment?

a. Oral temperature
b. Peripheral pulses
c. Extremity movement
d. Pupil reaction to light

A

c. Extremity movement

All patients with electrical burns should be considered at risk for cervical spine injury, and assessments of extremity movement will provide baseline data. The other assessment data are also necessary but not as essential as determining the cervical spine status.

159
Q

An employee spills industrial acids on both arms and legs at work. What is the priority action that the occupational health nurse at the facility should take?

a. Remove nonadherent clothing and watch.
b. Apply an alkaline solution to the affected area.
c. Place cool compresses on the area of exposure.
d. Cover the affected area with dry, sterile dressings.

A

a. Remove nonadherent clothing and watch.

With chemical burns, the initial action is to remove the chemical from contact with the skin as quickly as possible. Remove nonadherent clothing, shoes, watches, jewelry, glasses, or contact lenses (if face was exposed). Flush chemical from wound and surrounding area with copious amounts of saline solution or water. Covering the affected area or placing cool compresses on the area will leave the chemical in contact with the skin. Application of an alkaline solution is not recommended

160
Q

A patient who has burns on the arms, legs, and chest from a house fire has become agitated and restless 8 hours after being admitted to the hospital. Which action should the nurse take first?

a. Stay at the bedside and reassure the patient.
b. Administer the ordered morphine sulfate IV.
c. Assess orientation and level of consciousness.
d. Use pulse oximetry to check the oxygen saturation.

A

d.Use pulse oximetry to check the oxygen saturation.

Agitation in a patient who may have suffered inhalation injury might indicate hypoxia, and this should be assessed by the nurse first. Administration of morphine may be indicated if the nurse determines that the agitation is caused by pain. Assessing level of consciousness and orientation is also appropriate but not as essential as determining whether the patient is hypoxemic. Reassurance is not helpful to reduce agitation in a hypoxemic patient.

161
Q

A patient arrives in the emergency department with facial and chest burns caused by a house fire. Which action should the nurse take first?

a. Auscultate the patient’s lung sounds.
b. Determine the extent and depth of the burns.
c. Infuse the ordered lactated Ringer’s solution.
d. Administer the ordered hydromorphone (Dilaudid).

A

a. Auscultate the patient’s lung sounds.

A patient with facial and chest burns is at risk for inhalation injury, and assessment of airway and breathing is the priority. The other actions will be completed after airway management is assured.

162
Q

A patient with extensive electrical burn injuries is admitted to the emergency department. Which prescribed intervention should the nurse implement first?

a. Assess oral temperature.
b. Check a potassium level.
c. Place on cardiac monitor.
d. Assess for pain at contact points.

A

c. Place on cardiac monitor.

After an electrical burn, the patient is at risk for fatal dysrhythmias and should be placed on a cardiac monitor. Assessing the oral temperature is not as important as assessing for cardiac dysrhythmias. Checking the potassium level is important. However, it will take time before the laboratory results are back. The first intervention is to place the patient on a cardiac monitor and assess for dysrhythmias, so that they can be treated if occurring. A decreased or increased potassium level will alert the nurse to the possibility of dysrhythmias. The cardiac monitor will alert the nurse immediately of any dysrhythmias. Assessing for pain is important, but the patient can endure pain until the cardiac monitor is attached. Cardiac dysrhythmias can be lethal.

163
Q

The nurse is caring for a patient with superficial partial-thickness burns of the face sustained within the last 12 hours. Upon assessment the nurse would expect to find which manifestation?

a. Blisters
b. Reddening of the skin
c. Destruction of all skin layers
d. Damage to sebaceous glands

A

b. Reddening of the skin

The clinical appearance of superficial partial-thickness burns includes erythema, blanching with pressure, and pain and minimal swelling with no vesicles or blistering during the first 24 hours.

164
Q

Eight hours after a thermal burn covering 50% of a patient’s total body surface area (TBSA) the nurse assesses the patient. Which information would be a priority to communicate to the health care provider?

a. Blood pressure is 95/48 per arterial line.
b. Serous exudate is leaking from the burns.
c. Cardiac monitor shows a pulse rate of 108.
d. Urine output is 20 mL per hour for the past 2 hours.

A

d. Urine output is 20 mL per hour for the past 2 hours.

The urine output should be at least 0.5 to 1.0 mL/kg/hr during the emergent phase, when the patient is at great risk for hypovolemic shock. The nurse should notify the health care provider because a higher IV fluid rate is needed. BP during the emergent phase should be greater than 90 systolic, and the pulse rate should be less than 120. Serous exudate from the burns is expected during the emergent phase.

165
Q

Which patient should the nurse assess first?

a. A patient with smoke inhalation who has wheezes and altered mental status
b. A patient with full-thickness leg burns who has a dressing change scheduled
c. A patient with abdominal burns who is complaining of level 8 (0 to 10 scale) pain
d. A patient with 40% total body surface area (TBSA) burns who is receiving IV fluids at 500 mL/hour

A

a. A patient with smoke inhalation who has wheezes and altered mental status

This patient has evidence of lower airway injury and hypoxemia and should be assessed immediately to determine the need for oxygen or intubation. The other patients should also be assessed as rapidly as possible, but they do not have evidence of life-threatening complications.

166
Q

A patient who was found unconscious in a burning house is brought to the emergency department by ambulance. The nurse notes that the patient’s skin color is bright red. Which action should the nurse take first?

a. Insert two large-bore IV lines.
b. Check the patient’s orientation.
c. Assess for singed nasal hair and dark oral mucous membranes.
d. Place the patient on 100% oxygen using a non-rebreather mask.

A

d. Place the patient on 100% oxygen using a non-rebreather mask.

The patient’s history and skin color suggest carbon monoxide poisoning, which should be treated by rapidly starting oxygen at 100%. The other actions can be taken after the action to correct gas exchange.

167
Q

The nurse is reviewing laboratory results on a patient who had a large burn 48 hours ago. Which result requires priority action by the nurse?

a. Hematocrit 53%
b. Serum sodium 147 mEq/L
c. Serum potassium 6.1 mEq/L
d. Blood urea nitrogen 37 mg/dL

A

c. Serum potassium 6.1 mEq/L

Hyperkalemia can lead to fatal dysrhythmias and indicates that the patient requires cardiac monitoring and immediate treatment to lower the potassium level. The other laboratory values are also abnormal and require changes in treatment, but they are not as immediately life threatening as the elevated potassium level.

168
Q

The charge nurse observes the following actions being taken by a new nurse on the burn unit. Which action by the new nurse would require an intervention by the charge nurse?

a. The new nurse uses clean latex gloves when applying antibacterial cream to a burn wound.
b. The new nurse obtains burn cultures when the patient has a temperature of 95.2° F (35.1° C).
c. The new nurse administers PRN fentanyl (Sublimaze) IV to a patient 5 minutes before a dressing change.
d. The new nurse calls the health care provider for a possible insulin order when a nondiabetic patient’s serum glucose is elevated.

A

a. The new nurse uses clean latex gloves when applying antibacterial cream to a burn wound.

Sterile gloves should be worn when applying medications or dressings to a burn. Hypothermia is an indicator of possible sepsis, and cultures are appropriate. Nondiabetic patients may require insulin because stress and high calorie intake may lead to temporary hyperglycemia. Fentanyl peaks 5 minutes after IV administration, and should be used just before and during dressing changes for pain management.

169
Q

Which nursing action is a priority for a patient who has suffered a burn injury while working on an electrical power line?

a. Obtain the blood pressure.
b. Stabilize the cervical spine.
c. Assess for the contact points.
d. Check alertness and orientation.

A

b. Stabilize the cervical spine.

Cervical spine injuries are commonly associated with electrical burns. Therefore stabilization of the cervical spine takes precedence after airway management. The other actions are also included in the emergent care after electrical burns, but the most important action is to avoid spinal cord injury.

170
Q

Which action will the nurse include in the plan of care for a patient in the rehabilitation phase after a burn injury to the right arm and chest?

a. Keep the right arm in a position of comfort.
b. Avoid the use of sustained-release narcotics.
c. Teach about the purpose of tetanus immunization.
d. Apply water-based cream to burned areas frequently.

A

d. Apply water-based cream to burned areas frequently.

Application of water-based emollients will moisturize new skin and decrease flakiness and itching. To avoid contractures, the joints of the right arm should be positioned in an extended position, which is not the position of comfort. Patients may need to continue the use of opioids during rehabilitation. Tetanus immunization would have been given during the emergent phase of the burn injury.

171
Q

A young adult patient who is in the rehabilitation phase 6 months after a severe face and neck burn tells the nurse, “I’m sorry that I’m still alive. My life will never be normal again.” Which response by the nurse is best?

a. “Most people recover after a burn and feel satisfied with their lives.”
b. “It’s true that your life may be different. What concerns you the most?”
c. “It is really too early to know how much your life will be changed by the burn.”
d. “Why do you feel that way? You will be able to adapt as your recovery progresses.”

A

b. “It’s true that your life may be different. What concerns you the most?”

This response acknowledges the patient’s feelings and asks for more assessment data that will help in developing an appropriate plan of care to assist the patient with the emotional response to the burn injury. The other statements are accurate, but do not acknowledge the anxiety and depression that the patient is expressing.

172
Q

The nurse estimates the extent of a burn using the rule of nines for a patient who has been admitted with deep partial-thickness burns of the anterior trunk and the entire left arm. What percentage of the patient’s total body surface area (TBSA) has been injured?

A. 9%

B. 18%

C. 27%

D. 36%

E. 45%

A

C. 27%

When using the rule of nines, the anterior trunk is considered to cover 18% of the patient’s body and each arm is 9%.

173
Q

In caring for a patient with burns to the back, the nurse knows that the patient is moving out of the emergent phase of burn injury when what is observed?

a. Serum sodium and potassium increase.
b. Serum sodium and potassium decrease.
c. Edema and arterial blood gases improve.
d. Diuresis occurs and hematocrit decreases.

A

d. Diuresis occurs and hematocrit decreases.

In the emergent phase, the immediate, life-threatening problems from the burn, hypovolemic shock and edema, are treated and resolved. Toward the end of the emergent phase, fluid loss and edema formation end. Interstitial fluid returns to the vascular space and diuresis occurs. Urinary output is the most commonly used parameter to assess the adequacy of fluid resuscitation. The hemolysis of red blood cells (RBCs) and thrombosis of burned capillaries also decreases circulating RBCs. When the fluid balance has been restored, dilution causes the hematocrit levels to drop. Initially sodium moves to the interstitial spaces and remains there until edema formation ceases, so sodium levels increase at the end of the emergent phase as the sodium moves back to the vasculature. Initially potassium level increases as it is released from injured cells and hemolyzed RBCs so potassium levels decrease at the end of the emergent phase when fluid levels normalize.

174
Q

A patient is admitted to the burn unit with second- and third-degree burns covering the face, entire right upper extremity, and right anterior trunk area. Using the rule of nines, what should the nurse calculate the extent of these burns as being?

a. 18%
b. 22.5%
c. 27%
d. 36%

A

b. 22.5%

Using the rule of nines, for these second- and third-degree burns, the face encompasses 4.5% of the body area, the entire right arm encompasses 9% of the body area, and the entire anterior trunk encompasses 18% of the body area. Because the patient has burns on only the right side of the anterior trunk, the nurse would assess that burn as encompassing half of the 18%, or 9%. Therefore, adding the three areas together (4.5 + 9 + 9), the nurse would correctly calculate the extent of this patient’s burns to cover approximately 22.5% of the total body surface area.

175
Q

A patient with a burn inhalation injury is receiving albuterol for the treatment of bronchospasm. What is the most important adverse effect of this medication for the nurse to monitor?

a. Tachycardia
b. Restlessness
c. Hypokalemia
d. Gastrointestinal (GI) distress

A

a. Tachycardia

Albuterol stimulates ß-adrenergic receptors in the lungs to cause bronchodilation. However, it is a noncardioselective agent so it also stimulates the ß-receptors in the heart to increase the heart rate. Restlessness and GI upset may occur but will decrease with use. Hypokalemia does not occur with albuterol

176
Q

Which patient should the nurse prepare to transfer to a regional burn center?

a. A 25-yr-old pregnant patient with a carboxyhemoglobin level of 1.5%
b. A 39-yr-old patient with a partial-thickness burn to the right upper arm
c. A 53-yr-old patient with a chemical burn to the anterior chest and neck
d. A 42-yr-old patient who is scheduled for skin grafting of a burn wound

A

c. A 53-yr-old patient with a chemical burn to the anterior chest and neck

The American Burn Association (ABA) has established referral criteria to determine which burn injuries should be treated in burn centers where specialized facilities and personnel are available to handle this type of trauma. Patients with chemical burns should be referred to a burn center. A normal serum carboxyhemoglobin level for nonsmokers is 0% to 1.5% and for smokers is 4% to 9%. Skin grafting for burn wound management is not a criterion for a referral to a burn center. Partial-thickness burns greater than 10% total body surface area (TBSA) should be referred to a burn center. A burn to the right upper arm is 4% TBSA

177
Q

A patient is admitted to the emergency department with first- and second-degree burns after being involved in a house fire. Which assessment findings would alert the nurse to the presence of an inhalation injury (select all that apply.)?

Select all that apply.

a. Singed nasal hair
b. Generalized pallor
c. Painful swallowing
d. Burns on the upper extremities
e. History of being involved in a large fire

A

a. Singed nasal hair
b. Generalized pallor
c. Painful swallowing
e. History of being involved in a large fire

Reliable clues to the occurrence of inhalation injury is the presence of facial burns, singed nasal hair, hoarseness, painful swallowing, darkened oral and nasal membranes, carbonaceous sputum, history of being burned in an enclosed space, altered mental status, and “cherry red” skin color.

178
Q

A patient with type 2 diabetes mellitus is in the acute phase of burn care with electrical burns on the left side of the body and a serum glucose level of 485 mg/dL. What is the nurse’s priority intervention for this patient?

a. Replace the blood lost.
b. Maintain a neutral pH.
c. Maintain fluid balance.
d. Replace serum potassium.

A

c. Maintain fluid balance.

This patient is most likely experiencing hyperosmolar hyperglycemic syndrome (HHS). HHS dehydrates a patient rapidly. Thus HHS combined with the massive fluid losses of a burn tremendously increase this patient’s risk for hypovolemic shock and serious hypotension. This is clearly the nurse’s priority because the nurse must keep up with the patient’s fluid requirements to prevent circulatory collapse caused by low intravascular volume. There is no mention of blood loss. Fluid resuscitation will help to correct the pH and serum potassium abnormalities.

179
Q

Which of the following patients are at risk for developing Cushing’s Syndrome?

A. A patient with a tumor on the pituitary gland, which is causing too much ACTH to be secreted.

B. A patient taking glucocorticoids for several weeks.

C. A patient with a tuberculosis infection.

D. A patient who is post-opt from an adrenalectomy.

A

B. A patient taking glucocorticoids for several weeks.

Remember that CUSHING’S DISEASE is caused by the pituitary gland producing too much ACTH which in turn increases cortisol. Cushing’s SYNDROME is caused by medication therapy of glucocorticoids. An adrenalectomy is a treatment for Cushing’s Disease (so this is not the answer in this case) and TB is a risk factor for developing ADDISON’S Disease.

180
Q

Addison’s Disease is:

A. Increased secretion of cortisol

B. Increased secretion of aldosterone and cortisol

C. Decreased secretion of cortisol

D. Decreased secretion of aldosterone and cortisol

A

D. Decreased secretion of aldosterone and cortisol

181
Q

A patient with Addison’s Disease is being discharged home on Prednisone. Which of the following statements by the patient warrants you to re-educate the patient?

A. “I will notify the doctor if I become sick or experience extra stress.”

B. “I will take this medication as needed when symptoms present.”

C. “I will take this medication at the same time every day.”

D. “My daughter has bought me a Medic-Alert bracelet.”

A

B. “I will take this medication as needed when symptoms present.”

182
Q

A patient is admitted to the ER. The patient is unconscious on arrival. However, the patient’s family is with the patient and reports that before the patient became unconscious she was complaining of severe pain in the abdomen, legs, and back, and has been experiencing worsening confusion. In addition, they also report the patient has not been taking any medications. The patient was recently discharged from the hospital for treatment of low cortisol and aldosterone levels. On assessment, you note the patient’s blood pressure is 70/45. What disorder is this patient most likely experiencing?

A. Addisonian Crisis

B. Cushing Syndrome

C. Thyroid crisis

D. Hashimoto thyroiditis

A

A. Addisonian Crisis

Note the patient is experiencing the signs and symptoms of Addisonian Crisis. The red flag in this scenario are the patient’s symptoms, recent hospitalization diagnosis, and that she is not taking any medications. Remember that patients who have Addision’s disease are at risk for Addisonian Crisis, especially if they are not taking their prescribed hormone therapy replacement.

183
Q

A patient with Addison’s Disease should consume which of the following diets?

A. High fat and fiber

B. Low potassium and high protein

C. High protein, carbs, and adequate sodium

D. Low carbs, high protein, and increased sodium

A

C. High protein, carbs, and adequate sodium

184
Q

In Cushing’s Disease and Syndrome there are:

A. Increased cortisol production

B. Low potassium and glucose levels

C. Increased production of aldosterone and cortisol

D. Decreased production of cortisol and aldosterone

A

A. Increased cortisol production

185
Q

A patient with Cushing’s syndrome will be undergoing an adrenalectomy. Which of the following will be included in the patient’s discharge teaching after the procedure?

A. Glucocorticoid replacement therapy

B. Avoiding avocadoes and pears

C. Declomycin therapy

D. Signs and symptoms of Grave’s Disease

A

A. Glucocorticoid replacement therapy

186
Q

Which of the following is not a typical sign and symptom of Cushing’s Syndrome?

A. Hyperpigmentation of the skin

B. Hirsutism

C. Purplish striae

D. Moon Face

A

A. Hyperpigmentation of the skin

187
Q

In Cushing’s disease, the _______ is secreting too much ACTH (Adrenocorticotropic hormone) which is causing an increase in cortisol production.

A. Adrenal cortex

B. Pituitary gland

C. Thyroid gland

D. Hypothalamus

A

B. Pituitary gland

188
Q

The nurse is planning care for a 52-year-old male client in acute addisonian crisis. Which nursing diagnosis should receive the highest priority?

  1. Risk for Infection
  2. Decreased cardiac output
  3. Impaired physical mobility
  4. Imbalanced nutrition: less than body requirement
A
  1. Decreased cardiac output

An acute addisonian crisis is a life-threatening event, caused by deficiencies of cortisol and aldosterone. Glucocorticoid insufficiency causes a decrease in cardiac output and vascular tone, leading to hypovolemia. The client becomes tachycardic and hypotensive and may develop shock and circulatory collapse. The client with Addison’s disease is at risk for infection; however, reducing infection isn’t a priority during an addisonian crisis. Impaired physical mobility is also an appropriate nursing diagnosis for the client with Addison’s disease, but it isn’t a priority in a crisis. Imbalanced nutrition: Less than body requirements is also an important nursing diagnosis for the client with Addison’s disease but not a priority during a crisis.

189
Q

During the first 24 hours after a client is diagnosed with Addisonian crisis, which intervention should the nurse perform frequently?

  1. Weigh the client
  2. Test urine for ketones
  3. Assess vital signs
  4. Administer oral hydrocortisone
A
  1. Assess vital signs

Because the client in Addisonian crisis is unstable, vital signs and fluid and electrolyte balance should be assessed every 30 minutes until he’s stable. Daily weights are sufficient when assessing the client’s condition. The client shouldn’t have ketones in his urine, so there is no need to assess the urine for their presence. Oral hydrocortisone isn’t administered during the first 24 hours in severe adrenal insufficiency.

190
Q

A client is admitted to the health care facility for evaluation for Addison’s disease. Which laboratory test result best supports a diagnosis of Addison’s disease

  1. BUN level of 12 mg/dl
  2. Blood glucose level of 90 mg/dl
  3. Serum sodium level of 134 mEq/L
  4. Serum potassium level of 5.8 mEq/L
A
  1. Serum potassium level of 5.8 mEq/L

Addison’s disease decreases the production of aldosterone, cortisol, and androgen, causing urinary sodium and fluid losses, an increased serum potassium level, and hypoglycemia. Therefore, an elevated serum potassium level of 5.8 mEq/L best supports a diagnosis of Addison’s disease. A BUN level of 12 mg/dl and a blood glucose level of 90 mg/dl are within normal limits. In a client with Addison’s disease, the serum sodium level would be much lower than 134 mEq/L, a nearly normal level.

191
Q

When teaching a client with Cushing’s syndrome about dietary changes, the nurse should instruct the client to increase intake of:

  1. fresh fruits
  2. dairy products
  3. processed meats
  4. cereals and grains
A
  1. fresh fruits

Cushing’s syndrome causes sodium retention, which increases urinary potassium loss. Therefore, the nurse should advise the client to increase intake of potassium-rich foods, such as fresh fruit. The client should restrict consumption of dairy products, processed meats, cereals, and grains because they contain significant amounts of sodium.

192
Q

In a 28-year-old female client who is being successfully treated for Cushing’s syndrome, the nurse would expect a decline in:

  1. serum glucose level
  2. hair loss
  3. bone mineralization
  4. menstrual flow
A
  1. serum glucose level

Hyperglycemia, which develops from glucocorticoid excess, is a manifestation of Cushing’s syndrome. With successful treatment of the disorder, serum glucose levels decline. Hirsutism is common in Cushing’s syndrome; therefore, with successful treatment, abnormal hair growth also declines. Osteoporosis occurs in Cushing’s syndrome; therefore, with successful treatment, bone mineralization increases. Amenorrhea develops in Cushing’s syndrome. With successful treatment, the client experiences a return of menstrual flow, not a decline in it.

193
Q

A client with a history of Addison’s disease and flulike symptoms accompanied by nausea and vomiting over the past week is brought to the facility. When he awoke this morning, his wife noticed that he acted confused and was extremely weak. The client’s blood pressure is 90/58 mm Hg, his pulse is 116 beats/minute, and his temperature is 101° F (38.3° C). A diagnosis of acute adrenal

  1. Insulin
  2. Hydrocortisone
  3. Potassium
  4. Hypotonic saline
A
  1. Hydrocortisone

Emergency treatment for acute adrenal insufficiency (Addisonian crisis) is I.V. infusion of hydrocortisone and saline solution. The client is usually given a dose containing hydrocortisone 100 mg I.V. in normal saline every 6 hours until the client’s blood pressure returns to normal. Insulin isn’t indicated in this situation because adrenal insufficiency is usually associated with hypoglycemia. Potassium isn’t indicated because these clients are usually hyperkalemic. The client needs normal — not hypotonic — saline solution.

194
Q

The nurse is providing emergent care for a patient with a possible inhalation injury sustained in a house fire. The patient is anxious and disoriented, and the skin is a cherry red color. What is the priority action by the nurse?

a. Administer 100% humidified oxygen.
b. Teach the patient deep breathing exercises.
c. Encourage the patient to express his feelings.
d. Assist the patient to a high Fowler’s position.

A

a. Administer 100% humidified oxygen.

Carbon monoxide (CO) poisoning may occur in house fires. CO displaces oxygen on the hemoglobin molecule resulting in hypoxia. High levels of CO in the blood result in a skin color that is described as cherry red. Hypoxia may cause anxious behaviors and altered mental status. Emergency treatment for inhalation injury and CO poisoning includes the immediate administration of 100% humidified oxygen. The other interventions are appropriate for inhalation injury but are not as urgent as oxygen administration.

195
Q

The nurse is caring for a patient who sustained a deep partial-thickness burn to the anterior chest area during a workplace accident 6 hours ago. Which assessment findings would the nurse identify as congruent with this type of burn?

a. Skin is hard with a dry, waxy white appearance.
b. Skin is shiny and red with clear, fluid-filled blisters.
c. Skin is red and blanches when slight pressure is applied.
d. Skin is leathery with visible muscles, tendons, and bones.

A

b. Skin is shiny and red with clear, fluid-filled blisters.

Deep partial-thickness burns have fluid-filled vesicles that are red and shiny. They may appear wet (if vesicles have ruptured), and mild to moderate edema may be present. Superficial partial-thickness burns are red and blanch with pressure vesicles that appear 24 hours after the burn injury. Full-thickness burns are dry, waxy white, leathery, or hard, and there may be involvement of muscles, tendons, and bones.

196
Q

The nurse is planning care for a patient with partial- and full-thickness skin destruction related to burn injury of the lower extremities. Which interventions will the nurse include in this patient’s care (select all that apply.)?

a. Escharotomy
b. Administration of diuretics
c. IV and oral pain medications
d. Daily cleansing and debridement
e. Application of topical antimicrobial agent

A

a. Escharotomy
c. IV and oral pain medications
d. Daily cleansing and debridement
e. Application of topical antimicrobial agent.

An escharotomy (a scalpel incision through full-thickness eschar) is frequently required to restore circulation to compromised extremities. Daily cleansing and debridement as well as application of an antimicrobial ointment are expected interventions used to minimize infection and enhance wound healing. Pain control is essential in the care of a patient with a burn injury. With full-thickness burns, myoglobin and hemoglobin released into the bloodstream can occlude renal tubules. Adequate fluid replacement is used to prevent this occlusion.

197
Q

The nurse is caring for a 71-kg patient during the first 12 hours after a thermal burn injury. Which outcomes indicate adequate fluid resuscitation (select all that apply.)?

a. Urine output is 46 mL/hr.
b. Heart rate is 94 beats/min.
c. Urine specific gravity is 1.040.
d. Mean arterial pressure is 54 mm Hg.
e. Systolic blood pressure is 88 mm Hg.

A

a. Urine output is 46 mL/hr
b. Heart rate is 94 beats/min.

Assessment of the adequacy of fluid resuscitation is best made using either urine output or cardiac factors. Urine output should be 0.5 to 1 mL/kg/hr (or 75 to 100 mL/hr for an electrical burn patient with evidence of hemoglobinuria/myoglobinuria). Cardiac factors include a mean arterial pressure (MAP) greater than 65 mm Hg, systolic BP greater than 90 mm Hg, heart rate less than 120 beats/min. Normal range for urine specific gravity is 1.003 to 1.030.

198
Q

When caring for a patient with an electrical burn injury, which order from the health care provider should the nurse question?

a. Mannitol 75 g IV
b. Urine for myoglobulin
c. Lactated Ringer’s solution at 25 mL/hr
d. Sodium bicarbonate 24 mEq every 4 hours

A

c. Lactated Ringer’s solution at 25 mL/hr.

Electrical injury puts the patient at risk for myoglobinuria, which can lead to acute renal tubular necrosis (ATN). Treatment consists of infusing lactated Ringer’s solution at 2 to 4 mL/kg/%TBSA, a rate sufficient to maintain urinary output at 75 to 100 mL/hr. Mannitol can also be used to maintain urine output. Sodium bicarbonate may be given to alkalinize the urine. The urine would also be monitored for the presence of myoglobin. An infusion rate of 25 mL/hr is not sufficient to maintain adequate urine output in prevention and treatment of ATN.

199
Q
A
200
Q

The patient in the emergent phase of a burn injury is being treated for severe pain. What medication should the nurse anticipate administering to the patient?

a. Subcutaneous (SQ) tetanus toxoid
b. Intravenous (IV) morphine sulfate
c. Intramuscular (IM) hydromorphone
d. Oral oxycodone and acetaminophen

A

b. Intravenous (IV) morphine sulfate.

IV medications are used for burn injuries in the emergent phase to rapidly deliver relief and prevent unpredictable absorption as would occur with the IM route. The PO route is not used because GI function is slowed or impaired because of shock or paralytic ileus, although oxycodone and acetaminophen may be used later in the patient’s recovery. Tetanus toxoid may be administered but not for pain.

201
Q

A patient arrives in the emergency department after sustaining a full-thickness thermal burn to both arms while putting lighter fluid on a grill. What manifestations should the nurse expect?

a. Severe pain, blisters, and blanching with pressure
b. Pain, minimal edema, and blanching with pressure
c. Redness, evidence of inhalation injury, and charred skin
d. No pain, waxy white skin, and no blanching with pressure

A

d. No pain, waxy white skin, and no blanching with pressure

With full-thickness burns, the nerves and vasculature in the dermis are destroyed so there is no pain, the tissue is dry and waxy-looking or may be charred, and there is no blanching with pressure. Severe pain, blisters, and blanching occur with partial-thickness (deep, second-degree) burns. Pain, minimal edema, blanching, and redness occur with partial-thickness (superficial, first-degree) burns.

202
Q

The nurse is planning to change the dressing that covers a deep partial-thickness burn of the right lower leg. Which prescribed medication should the nurse administer to the patient 30 minutes before the scheduled dressing change?

a. Morphine
b. Sertraline
c. Zolpidem
d. Enoxaparin

A

a. Morphine.

Deep partial-thickness burns result in severe pain related to nerve injury. The nurse should plan to administer analgesics before the dressing change to promote patient comfort. Morphine is a common opioid used for pain control. Sedative/hypnotics and antidepressant agents also can be given with analgesics to control the anxiety, insomnia, and depression that patients may experience. Zolpidem promotes sleep. Sertraline is an antidepressant. Enoxaparin is an anticoagulant.

203
Q

The nurse is caring for a patient with superficial partial-thickness burns of the face sustained within the last 12 hours. Upon assessment the nurse would expect to find which manifestation?

a. Blisters
b. Reddening of the skin
c. Destruction of all skin layers
d. Damage to sebaceous glands

A

b. Reddening of the skin

The clinical appearance of superficial partial-thickness burns includes erythema, blanching with pressure, and pain and minimal swelling with no vesicles or blistering during the first 24 hours.

204
Q

A 78-kg patient with septic shock has a urine output of 30 mL/hr for the past 3 hours. The pulse rate is 120/minute and the central venous pressure and pulmonary artery wedge pressure are low. Which order by the health care provider will the nurse question?

a. Give PRN furosemide (Lasix) 40 mg IV.
b. Increase normal saline infusion to 250 mL/hr.
c. Administer hydrocortisone (Solu-Cortef) 100 mg IV.
d. Titrate norepinephrine (Levophed) to keep systolic BP >90 mm Hg.

A

a. Give PRN furosemide (Lasix) 40 mg IV.

Furosemide will lower the filling pressures and renal perfusion further for the patient with septic shock. The other orders are appropriate.

205
Q

A nurse is caring for a patient with shock of unknown etiology whose hemodynamic monitoring indicates BP 92/54, pulse 64, and an elevated pulmonary artery wedge pressure. Which collaborative intervention ordered by the health care provider should the nurse question?

a. Infuse normal saline at 250 mL/hr.
b. Keep head of bed elevated to 30 degrees.
c. Hold nitroprusside (Nipride) if systolic BP <90 mm Hg.
d. Titrate dobutamine (Dobutrex) to keep systolic BP >90 mm Hg.

A

a. Infuse normal saline at 250 mL/hr.

The patient’s elevated pulmonary artery wedge pressure indicates volume excess. A saline infusion at 250 mL/hr will exacerbate the volume excess. The other actions are appropriate for the patient.

206
Q

A 19-year-old patient with massive trauma and possible spinal cord injury is admitted to the emergency department (ED). Which assessment finding by the nurse will help confirm a diagnosis of neurogenic shock?

a. Inspiratory crackles.
b. Cool, clammy extremities.
c. Apical heart rate 45 beats/min.
d. Temperature 101.2° F (38.4° C).

A

c. Apical heart rate 45 beats/min.

Neurogenic shock is characterized by hypotension and bradycardia. The other findings would be more consistent with other types of shock

207
Q

An older patient with cardiogenic shock is cool and clammy and hemodynamic monitoring indicates a high systemic vascular resistance (SVR). Which intervention should the nurse anticipate doing next?

a. Increase the rate for the dopamine (Intropin) infusion.
b. Decrease the rate for the nitroglycerin (Tridil) infusion.
c. Increase the rate for the sodium nitroprusside (Nipride) infusion.
d. Decrease the rate for the 5% dextrose in normal saline (D5/.9 NS) infusion.

A

c. Increase the rate for the sodium nitroprusside (Nipride) infusion.

Nitroprusside is an arterial vasodilator and will decrease the SVR and afterload, which will improve cardiac output. Changes in the D5/.9 NS and nitroglycerin infusions will not directly decrease SVR. Increasing the dopamine will tend to increase SVR.

208
Q

After receiving 2 L of normal saline, the central venous pressure for a patient who has septic shock is 10 mm Hg, but the blood pressure is still 82/40 mm Hg. The nurse will anticipate an order for

a. nitroglycerine (Tridil).
b. norepinephrine (Levophed).
c. sodium nitroprusside (Nipride).
d. methylprednisolone (Solu-Medrol).

A

b. norepinephrine (Levophed).

When fluid resuscitation is unsuccessful, vasopressor drugs are administered to increase the systemic vascular resistance (SVR) and blood pressure, and improve tissue perfusion. Nitroglycerin would decrease the preload and further drop cardiac output and BP. Methylprednisolone (Solu-Medrol) is considered if blood pressure does not respond first to fluids and vasopressors. Nitroprusside is an arterial vasodilator and would further decrease SVR.

209
Q

To evaluate the effectiveness of the pantoprazole (Protonix) ordered for a patient with systemic inflammatory response syndrome (SIRS), which assessment will the nurse perform?

a. Auscultate bowel sounds.
b. Palpate for abdominal pain.
c. Ask the patient about nausea.
d. Check stools for occult blood.

A

d. Check stools for occult blood.

Proton pump inhibitors are given to decrease the risk for stress ulcers in critically ill patients. The other assessments also will be done, but these will not help in determining the effectiveness of the pantoprazole administration.

210
Q

A patient with cardiogenic shock has the following vital signs: BP 102/50, pulse 128, respirations 28. The pulmonary artery wedge pressure (PAWP) is increased and cardiac output is low. The nurse will anticipate an order for which medication?

a. 5% human albumin
b. Furosemide (Lasix) IV
c. Epinephrine (Adrenalin) drip
d. Hydrocortisone (Solu-Cortef)

A

b. Furosemide (Lasix) IV

The PAWP indicates that the patient’s preload is elevated, and furosemide is indicated to reduce the preload and improve cardiac output. Epinephrine would further increase heart rate and myocardial oxygen demand. 5% human albumin would also increase the PAWP. Hydrocortisone might be considered for septic or anaphylactic shock.

211
Q

The emergency department (ED) nurse receives report that a patient involved in a motor vehicle crash is being transported to the facility with an estimated arrival in 1 minute. In preparation for the patient’s arrival, the nurse will obtain

a. hypothermia blanket.
b. lactated Ringer’s solution.
c. two 14-gauge IV catheters.
d. dopamine (Intropin) infusion.

A

c. two 14-gauge IV catheters.

A patient with multiple trauma may require fluid resuscitation to prevent or treat hypovolemic shock, so the nurse will anticipate the need for 2 large bore IV lines to administer normal saline. Lactated Ringer’s solution should be used cautiously and will not be ordered until the patient has been assessed for possible liver abnormalities. Vasopressor infusion is not used as the initial therapy for hypovolemic shock. Patients in shock need to be kept warm not cool.

212
Q

Which finding is the best indicator that the fluid resuscitation for a patient with hypovolemic shock has been effective?

a. Hemoglobin is within normal limits.
b. Urine output is 60 mL over the last hour.
c. Central venous pressure (CVP) is normal.
d. Mean arterial pressure (MAP) is 72 mm Hg.9. Which finding is the best indicator that the fluid resuscitation for a patient with hypovolemic shock has been effective?

A

b. Urine output is 60 mL over the last hour.

Assessment of end organ perfusion, such as an adequate urine output, is the best indicator that fluid resuscitation has been successful. The hemoglobin level, CVP, and MAP are useful in determining the effects of fluid administration, but they are not as useful as data indicating good organ perfusion.

213
Q

Which intervention will the nurse include in the plan of care for a patient who has cardiogenic shock?

a. Check temperature every 2 hours.
b. Monitor breath sounds frequently.
c. Maintain patient in supine position.
d. Assess skin for flushing and itching.

A

b. Monitor breath sounds frequently.

Since pulmonary congestion and dyspnea are characteristics of cardiogenic shock, the nurse should assess the breath sounds frequently. The head of the bed is usually elevated to decrease dyspnea in patients with cardiogenic shock. Elevated temperature and flushing or itching of the skin are not typical of cardiogenic shock.

214
Q

Norepinephrine (Levophed) has been prescribed for a patient who was admitted with dehydration and hypotension. Which patient data indicate that the nurse should consult with the health care provider before starting the norepinephrine?

a. The patient’s central venous pressure is 3 mm Hg.
b. The patient is in sinus tachycardia at 120 beats/min.
c. The patient is receiving low dose dopamine (Intropin).
d. The patient has had no urine output since being admitted.

A

a. The patient’s central venous pressure is 3 mm Hg.

Adequate fluid administration is essential before administration of vasopressors to patients with hypovolemic shock. The patient’s low central venous pressure indicates a need for more volume replacement. The other patient data are not contraindications to norepinephrine administration.

215
Q

A nurse is assessing a patient who is receiving a nitroprusside (Nipride) infusion to treat cardiogenic shock. Which finding indicates that the medication is effective?

a. No new heart murmurs
b. Decreased troponin level
c. Warm, pink, and dry skin
d. Blood pressure 92/40 mm Hg

A

c. Warm, pink, and dry skin

Warm, pink, and dry skin indicates that perfusion to tissues is improved. Since nitroprusside is a vasodilator, the blood pressure may be low even if the medication is effective. Absence of a heart murmur and a decrease in troponin level are not indicators of improvement in shock.

216
Q

Which data collected by the nurse caring for a patient who has cardiogenic shock indicate that the patient may be developing multiple organ dysfunction syndrome (MODS)?

a. The patient’s serum creatinine level is elevated.
b. The patient complains of intermittent chest pressure.
c. The patient’s extremities are cool and pulses are weak.
d. The patient has bilateral crackles throughout lung fields.

A

a. The patient’s serum creatinine level is elevated.

The elevated serum creatinine level indicates that the patient has renal failure as well as heart failure. The crackles, chest pressure, and cool extremities are all consistent with the patient’s diagnosis of cardiogenic shock.

217
Q

A patient with septic shock has a BP of 70/46 mm Hg, pulse 136, respirations 32, temperature 104° F, and blood glucose 246 mg/dL. Which intervention ordered by the health care provider should the nurse implement first?

a. Give normal saline IV at 500 mL/hr.
b. Give acetaminophen (Tylenol) 650 mg rectally.
c. Start insulin drip to maintain blood glucose at 110 to 150 mg/dL.
d. Start norepinephrine (Levophed) to keep systolic blood pressure >90 mm Hg.

A

a. Give normal saline IV at 500 mL/hr.

Because of the low systemic vascular resistance (SVR) associated with septic shock, fluid resuscitation is the initial therapy. The other actions also are appropriate, and should be initiated quickly as well.

218
Q

When the nurse educator is evaluating the skills of a new registered nurse (RN) caring for patients experiencing shock, which action by the new RN indicates a need for more education?

a. Placing the pulse oximeter on the ear for a patient with septic shock
b. Keeping the head of the bed flat for a patient with hypovolemic shock
c. Increasing the nitroprusside (Nipride) infusion rate for a patient with a high SVR
d. Maintaining the room temperature at 66° to 68° F for a patient with neurogenic shock

A

d. Maintaining the room temperature at 66° to 68° F for a patient with neurogenic shock

Patients with neurogenic shock may have poikilothermia. The room temperature should be kept warm to avoid hypothermia. The other actions by the new RN are appropriate

219
Q

The nurse is caring for a patient who has septic shock. Which assessment finding is most important for the nurse to report to the health care provider?

a. Blood pressure (BP) 92/56 mm Hg
b. Skin cool and clammy
c. Oxygen saturation 92%
d. Heart rate 118 beats/minute

A

b. Skin cool and clammy

Because patients in the early stage of septic shock have warm and dry skin, the patient’s cool and clammy skin indicates that shock is progressing. The other information will also be reported, but does not indicate deterioration of the patient’s status.

220
Q

A patient is admitted to the emergency department (ED) for shock of unknown etiology. The first action by the nurse should be to

a. administer oxygen.
b. obtain a 12-lead electrocardiogram (ECG).
c. obtain the blood pressure.
d. check the level of consciousness.

A

a. administer oxygen.

The initial actions of the nurse are focused on the ABCs—airway, breathing, and circulation—and administration of oxygen should be done first. The other actions should be accomplished as rapidly as possible after oxygen administration.

221
Q

During change-of-shift report, the nurse is told that a patient has been admitted with dehydration and hypotension after having vomiting and diarrhea for 4 days. Which finding is most important for the nurse to report to the health care provider?

a. New onset of confusion
b. Heart rate 112 beats/minute
c. Decreased bowel sounds
d. Pale, cool, and dry extremities

A

a. New onset of confusion

The changes in mental status are indicative that the patient is in the progressive stage of shock and that rapid intervention is needed to prevent further deterioration. The other information is consistent with compensatory shock.

222
Q
A
223
Q

A patient who has been involved in a motor vehicle crash arrives in the emergency department (ED) with cool, clammy skin; tachycardia; and hypotension. Which intervention ordered by the health care provider should the nurse implement first?

a. Insert two large-bore IV catheters.
b. Initiate continuous electrocardiogram (ECG) monitoring.
c. Provide oxygen at 100% per non-rebreather mask.
d. Draw blood to type and crossmatch for transfusions.

A

c. Provide oxygen at 100% per non-rebreather mask.

The first priority in the initial management of shock is maintenance of the airway and ventilation. ECG monitoring, insertion of IV catheters, and obtaining blood for transfusions should also be rapidly accomplished but only after actions to maximize oxygen delivery have been implemented.

224
Q

The patient with neurogenic shock is receiving a phenylephrine (Neo-Synephrine) infusion through a right forearm IV. Which assessment finding obtained by the nurse indicates a need for immediate action?

a. The patient’s heart rate is 58 beats/minute.
b. The patient’s extremities are warm and dry.
c. The patient’s IV infusion site is cool and pale.
d. The patient’s urine output is 28 mL over the last hour.

A

c. The patient’s IV infusion site is cool and pale.

The coldness and pallor at the infusion site suggest extravasation of the phenylephrine. The nurse should discontinue the IV and, if possible, infuse the medication into a central line. An apical pulse of 58 is typical for neurogenic shock but does not indicate an immediate need for nursing intervention. A 28-mL urinary output over 1 hour would require the nurse to monitor the output over the next hour, but an immediate change in therapy is not indicated. Warm, dry skin is consistent with early neurogenic shock, but it does not indicate a need for a change in therapy or immediate action.

225
Q

The following interventions are ordered by the health care provider for a patient who has respiratory distress and syncope after eating strawberries. Which will the nurse complete first?

a. Start a normal saline infusion.
b. Give epinephrine (Adrenalin).
c. Start continuous ECG monitoring.
d. Give diphenhydramine (Benadryl).

A

b. Give epinephrine (Adrenalin).

Epinephrine rapidly causes peripheral vasoconstriction, dilates the bronchi, and blocks the effects of histamine and reverses the vasodilation, bronchoconstriction, and histamine release that cause the symptoms of anaphylaxis. The other interventions are also appropriate but would not be the first ones completed.

226
Q

Which finding about a patient who is receiving vasopressin (Pitressin) to treat septic shock is most important for the nurse to communicate to the health care provider?

a. The patient’s urine output is 18 mL/hr.
b. The patient’s heart rate is 110 beats/minute.
c. The patient is complaining of chest pain.
d. The patient’s peripheral pulses are weak.

A

c. The patient is complaining of chest pain.

Because vasopressin is a potent vasoconstrictor, it may decrease coronary artery perfusion. The other information is consistent with the patient’s diagnosis and should be reported to the health care provider but does not indicate a need for a change in therapy.

227
Q
A
228
Q

After change-of-shift report in the progressive care unit, who should the nurse care for first?

a. Patient who had an inferior myocardial infarction 2 days ago and has crackles in the lung bases
b. Patient with suspected urosepsis who has new orders for urine and blood cultures and antibiotics
c. Patient who had a T5 spinal cord injury 1 week ago and currently has a heart rate of 54 beats/minute
d. Patient admitted with anaphylaxis 3 hours ago who now has clear lung sounds and a blood pressure of 108/58 mm Hg

A

b. Patient with suspected urosepsis who has new orders for urine and blood cultures and antibiotics

Antibiotics should be given within the first hour for patients who have sepsis or suspected sepsis in order to prevent progression to systemic inflammatory response syndrome and septic shock. The data on the other patients indicate that they are more stable. Crackles heard only at the lung bases do not require immediate intervention in a patient who has had a myocardial infarction. Mild bradycardia does not usually require atropine in patients who have a spinal cord injury. The findings for the patient admitted with anaphylaxis indicate resolution of bronchospasm and hypotension.

229
Q

A patient with suspected neurogenic shock after a diving accident has arrived in the emergency department. A cervical collar is in place. Which actions should the nurse take (select all that apply)?

a. Prepare to administer atropine IV.
b. Obtain baseline body temperature.
c. Infuse large volumes of lactated Ringer’s solution.
d. Provide high-flow oxygen (100%) by non-rebreather mask.
e. Prepare for emergent intubation and mechanical ventilation.

A

a. Prepare to administer atropine IV.
b. Obtain baseline body temperature.
d. Provide high-flow oxygen (100%) by non-rebreather mask.
e. Prepare for emergent intubation and mechanical ventilation.

All of the actions are appropriate except to give large volumes of lactated Ringer’s solution. The patient with neurogenic shock usually has a normal blood volume, and it is important not to volume overload the patient. In addition, lactated Ringer’s solution is used cautiously in all shock situations because the failing liver cannot convert lactate to bicarbonate.

230
Q

Which preventive actions by the nurse will help limit the development of systemic inflammatory response syndrome (SIRS) in patients admitted to the hospital (select all that apply)?

a. Use aseptic technique when caring for invasive lines or devices.
b. Ambulate postoperative patients as soon as possible after surgery.
c. Remove indwelling urinary catheters as soon as possible after surgery.
d. Advocate for parenteral nutrition for patients who cannot take oral feedings.
e. Administer prescribed antibiotics within 1 hour for patients with possible sepsis.

A

a. Use aseptic technique when caring for invasive lines or devices.
b. Ambulate postoperative patients as soon as possible after surgery.
c. Remove indwelling urinary catheters as soon as possible after surgery.
e. Administer prescribed antibiotics within 1 hour for patients with possible sepsis.

Because sepsis is the most frequent etiology for SIRS, measures to avoid infection such as removing indwelling urinary catheters as soon as possible, use of aseptic technique, and early ambulation should be included in the plan of care. Adequate nutrition is important in preventing SIRS. Enteral, rather than parenteral, nutrition is preferred when patients are unable to take oral feedings because enteral nutrition helps maintain the integrity of the intestine, thus decreasing infection risk. Antibiotics should be administered within 1 hour after being prescribed to decrease the risk of sepsis progressing to SIRS.

231
Q

To evaluate the effectiveness of fluid therapy, which physiological parameters would be most important for the nurse to assess?

a. Breath sounds and capillary refill
b. Blood pressure and oral temperature
c. Oral temperature and capillary refill
d. Right atrial pressure and urine output

A

d. Right atrial pressure and urine output

Early goal-directed therapy includes administration of IV fluids to keep central venous pressure at 8 mm Hg or greater. Combined with urine output, fluid therapy effectiveness can be adequately assessed. Evaluation of breath sounds assists with determining fluid overload in a patient but does not evaluate the effectiveness of fluid therapy. Capillary refill provides a quick assessment of the patient’s overall cardiovascular status, but this assessment is not reliable in a patient who is hypothermic or has peripheral circulatory problems. Evaluation of oral temperature does not assess the effectiveness of fluid therapy in patients in shock.

232
Q

The nurse is caring for a patient in the early stages of septic shock. The patient is slightly confused and flushed, with bounding peripheral pulses. Which hemodynamic values is the nurse most likely to assess?

a. High pulmonary artery occlusive pressure and high cardiac output
b. High systemic vascular resistance and low cardiac output
c. Low pulmonary artery occlusive pressure and low cardiac output
d. Low systemic vascular resistance and high cardiac output

A

d. Low systemic vascular resistance and high cardiac output

As a consequence of the massive vasodilation associated with septic shock, in the early stages, cardiac output is high with low systemic vascular resistance. In septic shock, pulmonary artery occlusion pressure is not elevated. In the early stages of septic shock, systemic vascular resistance is low and cardiac output is high. In the early stages of septic shock, cardiac output is high.

233
Q

The nurse is caring for a patient admitted with cardiogenic shock. Hemodynamic readings obtained with a pulmonary artery catheter include a pulmonary artery occlusion pressure (PAOP) of 18 mm Hg and a cardiac index (CI) of 1.0 L/min/m2. What is the priority pharmacological intervention?

a. Dobutamine
b. Furosemide
c. Phenylephrine
d. Sodium nitroprusside

A

a. Dobutamine

A high PAOP and a low cardiac index are findings consistent with cardiogenic shock. Positive inotropic agents (e.g., dobutamine) are given to increase the contractile force of the heart. As contractility increases, cardiac output and index increase and improve tissue perfusion. Administration of furosemide will assist only in managing fluid volume overload. Phenylephrine administration enhances vasoconstriction, which may increase afterload and further reduce cardiac output. Sodium nitroprusside is given to reduce afterload. There is no evidence to support a need for afterload reduction in this scenario.

234
Q

Ten minutes following administration of an antibiotic, the nurse assesses a patient to have edematous lips, hoarseness, and expiratory stridor. Vital signs assessed by the nurse include blood pressure 70/40 mm Hg, heart rate 130 beats/min, and respirations 36 breaths/min. What is the priority intervention?

a. Diphenhydramine 50 mg intravenously
b. Epinephrine 3 to 5 mL of a 1:10,000 solution intravenously
c. Methylprednisolone 125 mg intravenously
d. Ranitidine 50 mg intravenously

A

b. Epinephrine 3 to 5 mL of a 1:10,000 solution intravenously

The patient is exhibiting signs of anaphylaxis. For anaphylaxis with hypotension, epinephrine 0.3 to 0.5 mg (3 to 5 mL of 1:10,000 solution) is administered intravenously. Diphenhydramine will help block histamine release, but epinephrine is the drug of choice for anaphylaxis with hypotension. Corticosteroids, such as methylprednisolone, are used to reduce inflammation, but epinephrine is the drug of choice for anaphylaxis with hypotension. Ranitidine will help block histamine release, but epinephrine is the drug of choice for anaphylaxis with hypotension.

235
Q

While monitoring a patient for signs of shock, the nurse understands which system assessment to be of priority?

a. Central nervous system
b. Gastrointestinal system
c. Renal system
d. Respiratory system

A

a. Central nervous system

The central nervous system experiences decreased perfusion first. The patient will have central nervous system changes early during the course of shock, such as changes in the level of consciousness. Although the gastrointestinal, renal, and respiratory systems also experience changes during shock, changes in the central nervous system provide the earliest indication of decreased perfusion.

236
Q

The nurse has just completed administration of a 500 mL bolus of 0.9% normal saline in a patient with hypovolemic shock. The nurse assesses the patient to be slightly confused, with a mean arterial blood pressure (MAP) of 50 mm Hg, a heart rate of 110 beats/min, urine output of 10 mL for the past hour, and a central venous pressure (CVP/RAP) of 3 mm Hg. What is the best interpretation of these results by the nurse?

a. Patient response to therapy is appropriate.
b. Additional interventions are indicated.
c. More time is needed to assess response.
d. Values are normal for the patient condition.

A

b. Additional interventions are indicated.

Assessed vital signs and hemodynamic values indicate decreased circulating volume. The patient has not responded appropriately to therapy aimed at increasing circulating volume. Additional intervention is needed because response to therapy is not appropriate, values are abnormal, and timely intervention is critical for a patient with low circulating blood volume.

237
Q

The emergency department nurse admits a patient following a motor vehicle collision. Vital signs include blood pressure 70/50 mm Hg, heart rate 140 beats/min, respiratory rate 36 breaths/min, temperature 101° F and oxygen saturation (SpO2) 95% on 3 L of oxygen per nasal cannula. Laboratory results include hemoglobin 6.0 g/dL, hematocrit 20%, and potassium 4.0 mEq/L. Based on this assessment, what is most important for the nurse to include in the patient’s plan of care?

a. Insertion of an 18-gauge peripheral intravenous line
b. Application of cushioned heel protectors
c. Implementation of fall precautions
d. Implementation of universal precautions

A

a. Insertion of an 18-gauge peripheral intravenous line

Given the patient’s diagnosis, laboratory results, and supporting vital signs, restoring circulating blood volume is a priority and can be accomplished following insertion of an appropriate gauge IV (18) to facilitate blood and fluid administration. Universal precautions, fall precautions, and application of heel protectors are appropriate interventions but are not the immediate priority.

238
Q

The nurse is starting to administer a unit of packed red blood cells (PRBCs) to a patient admitted in hypovolemic shock secondary to hemorrhage. Vital signs include blood pressure 60/40 mm Hg, heart rate 150 beats/min, respirations 42 breaths/min, and temperature 100.6° F. What is the best action by the nurse?

a. Administer blood transfusion over at least 4 hours.
b. Notify the physician of the elevated temperature.
c. Titrate rate of blood administration to patient response.
d. Notify the physician of the patient’s heart rate.

A

c. Titrate rate of blood administration to patient response.

Given the acute nature of the patient’s blood loss, the nurse should titrate the rate of the blood transfusion to an improvement in the patient’s blood pressure. Administering the transfusion over 4 hours can lead to a prolonged state of hypoperfusion and end-organ damage. The heart rate will normalize as circulating blood volume is restored. A mildly elevated temperature does not take priority over restoring circulating blood volume.

239
Q

The nurse is caring for a patient in septic shock. The nurse assesses the patient to have a blood pressure of 105/60 mm Hg, heart rate 110 beats/min, respiratory rate 32 breaths/min, oxygen saturation (SpO2) 95% on 45% supplemental oxygen via Venturi mask, and a temperature of 102° F. The physician orders stat administration of an antibiotic. Which additional physician order should the nurse complete first?

a. Blood cultures
b. Chest x-ray
c. Foley insertion
d. Serum electrolytes

A

a. Blood cultures

Timely identification of the causative organism through blood cultures and the initiation of appropriate antibiotics following obtaining blood cultures improve the survival of patients with sepsis or septic shock. A chest x-ray, Foley insertion, and measurement of serum electrolytes may be included in the plan of care but are not the priority in this scenario.

240
Q

The nurse is caring for a patient admitted to the critical care unit 48 hours ago with a diagnosis of severe sepsis. As part of this patient’s care plan, what intervention is most important for the nurse to discuss with the multidisciplinary care team?

a. Frequent turning
b. Monitoring intake and output
c. Enteral feedings
d. Pain management

A

c. Enteral feedings

Initiation of enteral feedings within 24 to 48 hours of admission is critical in reducing the risk of infection by assisting in maintaining the integrity of the intestinal mucosa. Monitoring intake and output, frequent turning, and pain management are important aspects of care but are not a critical priority during the first 24 to 48 hours following admission.

241
Q

The nurse is caring for a 70-kg patient in hypovolemic shock. Upon initial assessment, the nurse notes a blood pressure of 90/50 mm Hg, heart rate 125 beats/min, respirations 32 breaths/min, central venous pressure (CVP/RAP) of 3 mm Hg, and urine output of 5 mL during the past hour. Following physician rounds, the nurse reviews the orders and questions which order?

a. Administer acetaminophen 650-mg suppository prn every 6 hours for pain.
b. Titrate dopamine intravenously for blood pressure less than 90 mm Hg systolic.
c. Complete neurological assessment every 4 hours for the next 24 hours.
d. Administer furosemide 20 mg IV every 4 hours for a CVP greater than or equal to 20 mm Hg.

A

b. Titrate dopamine intravenously for blood pressure less than 90 mm Hg systolic.

Vasoconstrictive agents should not be administered for hypotension in the presence of circulation fluid volume deficit, which this patient displays. The nurse should question the use of the dopamine infusion. All other listed orders are appropriate and have potential for use in the treatment of a hypovolemic shock.

242
Q

The nurse is administering intravenous norepinephrine at 5 mcg/kg/min via a 20-gauge peripheral intravenous (IV) catheter. Which assessment finding requires immediate action by the nurse?

a. Blood pressure 100/60 mm Hg
b. Swelling at the IV site
c. Heart rate of 110 beats/min
d. Central venous pressure (CVP) of 8 mm Hg

A

b. Swelling at the IV site

Swelling at the IV site is indicative of infiltration. Infusion of norepinephrine through an infiltrated IV site can lead to tissue necrosis and requires immediate intervention by the nurse. A blood pressure of 100/60 mm Hg, heart rate of 110 beats/min, and a CVP of 8 mm Hg are adequate and do not require immediate intervention.

243
Q

The nurse is caring for a patient in cardiogenic shock experiencing chest pain. Hemodynamic values assessed by the nurse include a cardiac index (CI) of 2.5 L/min/m2, heart rate of 70 beats/min, and a systemic vascular resistance (SVR) of 2200 dynes/sec/cm5. Upon review of physician orders, which order is most appropriate for the nurse to initiate?

a. Furosemide 20 mg intravenous (IV) every 4 hours as needed for CVP greater than or equal to 20 mm Hg
b. Nitroglycerin infusion titrated at a rate of 5 to 10 mcg/min as needed for chest pain
c. Dobutamine infusion at a rate of 2 to 20 mcg/kg/min as needed for CI less than 2 L/min/m2
d. Dopamine infusion at a rate of 5 to 10 mcg/kg/min to maintain a systolic BP of at least 90 mm Hg

A

b. Nitroglycerin infusion titrated at a rate of 5 to 10 mcg/min as needed for chest pain

The patient is complaining of chest pain and has an elevated systemic vascular resistance (SVR). To reduce afterload, ease the workload of the heart, and dilate the coronary arteries, improving oxygenation to the heart muscle, initiation of a nitroglycerin infusion is most appropriate. Assessment data do not support the initiation of other listed physician order options.

244
Q

The nurse is caring for a patient admitted with the early stages of septic shock. The nurse assesses the patient to be tachypneic, with a respiratory rate of 32 breaths/min. Arterial blood gas values assessed on admission are pH 7.50, CO2 28 mm Hg, HCO3 26. Which diagnostic study result reviewed by the nurse indicates progression of the shock state?

a. pH 7.40, CO2 40, HCO3 24
b. pH 7.45, CO2 45, HCO3 26
c. pH 7.35, CO2 40, HCO3 22
d. pH 7.30, CO2 45, HCO3 18

A

d. pH 7.30, CO2 45, HCO3 18

As shock progresses along the continuum, acidosis ensues, caused by metabolic acidosis, hypoxia, and anaerobic metabolism. A pH 7.30, CO2 45 mm Hg, HCO3 18 indicates metabolic acidosis and progression to a late stage of shock. All other listed arterial blood gas values are within normal limits.

245
Q

The nurse is caring for a patient admitted following a motor vehicle crash. Over the past 2 hours, the patient has received 6 units of packed red blood cells and 4 units of fresh frozen plasma by rapid infusion. To prevent complications, what is the priority nursing intervention?

a. Administer pain medication.
b. Turn patient every 2 hours.
c. Assess core body temperature.
d. Apply bilateral heel protectors.

A

c. Assess core body temperature.

Hypothermia is anticipated during the rapid infusion of fluids or blood products. Assessment of core body temperature is a priority. While administration of pain management, repositioning the patient every 2 hours, and application of heel protectors should be part of the patient care, given the rapid transfusion of blood products, these interventions are not the priority in this scenario.

246
Q

Fifteen minutes after beginning a transfusion of O negative blood to a patient in shock, the nurse assesses a drop in the patient’s blood pressure to 60/40 mm Hg, heart rate 135 beats/min, respirations 40 breaths/min, and a temperature of 102° F. The nurse notes the new onset of hematuria in the patient’s Foley catheter. What are the priority nursing actions? (Select all that apply.)

a. Administer acetaminophen.
b. Document the patient’s response.
c. Increase the rate of transfusion.
d. Notify the blood bank.
e. Notify the provider.
f. Stop the transfusion.

A

d. Notify the blood bank.
e. Notify the provider.
f. Stop the transfusion.

In the event of a reaction, the transfusion is stopped, the patient is assessed, and both the physician and laboratory are notified. All transfusion equipment (bag, tubing, and remaining solutions) and any blood or urine specimens obtained are sent to the laboratory according to hospital policy. The events of the reaction, interventions used, and patient response to treatment are documented although this occurs after immediate action has been taken. Acetaminophen is not warranted in the immediate recognition and treatment of a transfusion reaction. The infusion must be stopped. Increasing the infusion further increases the likelihood of worsening the transfusion reaction.

247
Q

The nurse is caring for a patient admitted with shock. The nurse understands which assessment findings best assess tissue perfusion in a patient in shock? (Select all that apply.)

a. Blood pressure
b. Heart rate
c. Level of consciousness
d. Pupil response
e. Respirations
f. Urine output

A

a. Blood pressure
c. Level of consciousness
f. Urine output

The level of consciousness assesses cerebral perfusion, urine output assesses renal perfusion, and blood pressure is a general indicator of systemic perfusion. Heart rate is not an indicator of perfusion. Pupillary response and respirations do not assess perfusion.

248
Q

The nurse is concerned that a client is demonstrating early signs of hypovolemic shock. Which assessment findings support the nurse’s concern?

Select all that apply.

A) Slight increase in pulse

B) Prolonged capillary refill time

C) Rapid weak pulse

D) Normal respirations

E) Normal blood pressure

A

A) Slight increase in pulse

B) Prolonged capillary refill time

D) Normal respirations

E) Normal blood pressure

Explanation: Manifestations of early hypovolemic shock include a slight increase in pulse, normal respirations, prolonged capillary refill time, and normal blood pressure. A weak rapid pulse is a characteristic of the irreversible stage of hypovolemic shock.

249
Q

An older adult client is experiencing hypovolemic shock. Which is the priority intervention for this client?

A) Administering analgesics for control of pain

B) Assessing the cause of bleeding

C) Providing replacement of volume

D) Establishing invasive cardiac monitoring

A

D) Establishing invasive cardiac monitoring

Explanation: With aging, there is a decrease in cardiac sympathetic activity. Older clients can have secondary volume depletion because of diuretics or malnutrition, and if prescribed a beta blocker, tachycardia may not occur as an early sign of hypovolemic shock. The older client will require early invasive monitoring in order to avoid excessive or inadequate volume restoration. This should be done early in the treatment phase. Replacement of volume would occur after invasive cardiac monitoring is established. Pain would be a consideration but would not be attended to as a first priority. Assessing the cause of bleeding would also occur after establishing invasive cardiac monitoring.

250
Q

A client is receiving intravenous nitroprusside (Nipride) for shock. Which adverse reactions will the nurse assess this client for when administering the infusion?

Select all that apply.

A) Muscle spasms

B) Tachycardia

C) Confusion

D) Gastrointestinal bleeding

E) Disorientation

A

A) Muscle spasms

B) Tachycardia

C) Confusion

E) Disorientation

Explanation: Nausea, muscle spasms, and disorientation are signs of thiocyanate poisoning, which can occur if the infusion is longer than 72 hours. Confusion, dizziness, and tachycardia are adverse reactions that the nurse should report immediately to the healthcare provider. Gastrointestinal bleeding is not an adverse effect of this medication.

251
Q

A nurse working in the intensive care unit (ICU) is caring for a client in progressive hemorrhagic shock. When planning care for this client, which does the nurse anticipate?

A) A sustained decrease of 10mmHg of the client’s mean arterial pressure (MAP)

B) A blood loss of 25%

C) A change from aerobic to anaerobic metabolism

D) A decrease in hydrostatic pressure within the capillary, shifting fluid into the interstitial space

A

C) A change from aerobic to anaerobic metabolism

Explanation: In intermediate or progressive hemorrhagic shock, there is a change from aerobic to anaerobic metabolism due to cellular hypoxia from decreased perfusion. This stage of shock occurs when there is sustained decrease of 20 mmHg or more of the client’s MAP and a blood loss of 35% to 50%. The acid by-products of anaerobic metabolism causes an increase, not decrease, in hydrostatic pressure within the capillary, shifting fluid into the interstitial space.

252
Q

A nurse is caring for a client after surgery. The client’s respiratory rate has increased from 12 to 18 breaths/min and the pulse rate increased from 86 to 98 beats/min since they were last assessed 4 hours ago. What action by the nurse is best?

a. Ask if the client needs pain medication.
b. Assess the client’s tissue perfusion further.
c. Document the findings in the client’s chart.
d. Increase the rate of the client’s IV infusion.

A

b. Assess the client’s tissue perfusion further.

Signs of the earliest stage of shock are subtle and may manifest in slight increases in heart rate, respiratory rate, or blood pressure. Even though these readings are not out of the normal range, the nurse should conduct a thorough assessment of the client, focusing on indicators of perfusion. The client may need pain medication, but this is not the priority at this time. Documentation should be done thoroughly but is not the priority either. The nurse should not increase the rate of the IV infusion without an order.

253
Q

The nurse gets the hand-off report on four clients. Which client should the nurse assess first?

a. Client with a blood pressure change of 128/74 to 110/88 mm Hg
b. Client with oxygen saturation unchanged at 94%
c. Client with a pulse change of 100 to 88 beats/min
d. Client with urine output of 40 mL/hr for the last 2 hours

A

a. Client with a blood pressure change of 128/74 to 110/88 mm Hg

This client has a falling systolic blood pressure, rising diastolic blood pressure, and narrowing pulse pressure, all of which may be indications of the progressive stage of shock. The nurse should assess this client first. The client with the unchanged oxygen saturation is stable at this point. Although the client with a change in pulse has a slower rate, it is not an indicator of shock since the pulse is still within the normal range; it may indicate the client’s pain or anxiety has been relieved, or he or she is sleeping or relaxing. A urine output of 40 mL/hr is only slightly above the normal range, which is 30 mL/hr.

254
Q

A nurse is caring for a client after surgery who is restless and apprehensive. The unlicensed assistive personnel (UAP) reports the vital signs and the nurse sees they are only slightly different from previous readings. What action does the nurse delegate next to the UAP?

a. Assess the client for pain or discomfort.
b. Measure urine output from the catheter.
c. Reposition the client to the unaffected side.
d. Stay with the client and reassure him or her.

A

b. Measure urine output from the catheter.

Urine output changes are a sensitive early indicator of shock. The nurse should delegate emptying the urinary catheter and measuring output to the UAP as a baseline for hourly urine output measurements. The UAP cannot assess for pain. Repositioning may or may not be effective for decreasing restlessness, but does not take priority over physical assessments. Reassurance is a therapeutic nursing action, but the nurse needs to do more in this situation.

255
Q

A client is in shock and the nurse prepares to administer insulin for a blood glucose reading of 208 mg/dL. The spouse asks why the client needs insulin as the client is not a diabetic. What response by the nurse is best?

a. High glucose is common in shock and needs to be treated.
b. Some of the medications we are giving are to raise blood sugar.
c. The IV solution has lots of glucose, which raises blood sugar.
d. The stress of this illness has made your spouse a diabetic.

A

a. High glucose is common in shock and needs to be treated.

High glucose readings are common in shock, and best outcomes are the result of treating them and maintaining glucose readings in the normal range. Medications and IV solutions may raise blood glucose levels, but this is not the most accurate answer. The stress of the illness has not made the client diabetic.

256
Q

A client arrives in the emergency department after being in a car crash with fatalities. The client has a nearly amputated leg that is bleeding profusely. What action by the nurse takes priority?

a. Apply direct pressure to the bleeding.
b. Ensure the client has a patent airway.
c. Obtain consent for emergency surgery.
d. Start two large-bore IV catheters.

A

b. Ensure the client has a patent airway.

Airway is the priority, followed by breathing and circulation (IVs and direct pressure). Obtaining consent is done by the physician.

257
Q

A client in shock is apprehensive and slightly confused. What action by the nurse is best?

a. Offer to remain with the client for awhile.
b. Prepare to administer antianxiety medication.
c. Raise all four siderails on the client’s bed.
d. Tell the client everything possible is being done.

A

a. Offer to remain with the client for awhile.

The nurses presence will be best to reassure this client. Antianxiety medication is not warranted as this will lower the clients blood pressure. Using all four side-rails on a hospital bed is considered a restraint in most facilities, although the nurse should ensure the client’s safety. Telling a confused client that everything is being done is not the most helpful response.

258
Q

A client is being discharged home after a large myocardial infarction and subsequent coronary artery bypass grafting surgery. The client’s sternal wound has not yet healed. What statement by the client most indicates a higher risk of developing sepsis after discharge?

a. All my friends and neighbors are planning a party for me.
b. I hope I can get my water turned back on when I get home.
c. I am going to have my daughter scoop the cat litter box.
d. My grandkids are so excited to have me coming home!

A

b. I hope I can get my water turned back on when I get home.

~ All these statements indicate a potential for leading to infection once the client gets back home. A large party might include individuals who are themselves ill and contagious. Having litter boxes in the home can expose the client to microbes that can lead to infection. Small children often have upper respiratory infections and poor hand hygiene that spread germs. However, the most worrisome statement is the lack of running water for handwashing and general hygiene and cleaning purposes.

259
Q

The student nurse studying shock understands that the common manifestations of this condition are directly related to which problems? (SATA)

a. Anaerobic metabolism
b. Hyperglycemia
c. Hypotension
d. Impaired renal perfusion
e. Increased perfusion

A

a. Anaerobic metabolism
c. Hypotension

The common manifestations of shock, no matter the cause, are directly related to the effects of anaerobic metabolism and hypotension. Hyperglycemia, impaired renal function, and increased perfusion are not manifestations of shock.

260
Q

The nurse caring for hospitalized clients includes which actions on their care plans to reduce the possibility of the clients developing shock? (SATA)

a. Assessing and identifying clients at risk
b. Monitoring the daily white blood cell count
c. Performing proper hand hygiene
d. Removing invasive lines as soon as possible
e. Using aseptic technique during procedures

A

a. Assessing and identifying clients at risk
c. Performing proper hand hygiene
d. Removing invasive lines as soon as possible
e. Using aseptic technique during procedures

Assessing and identifying clients at risk for shock is probably the most critical action the nurse can take to prevent shock from occurring. Proper hand hygiene, using aseptic technique, and removing IV lines and catheters are also important actions to prevent shock. Monitoring laboratory values does not prevent shock but can indicate a change.

261
Q

The nurse is caring for a patient in the early stages of septic shock. The patient is slightly confused and flushed, with bounding peripheral pulses. Which hemodynamic values is the nurse most likely to assess?

a. High pulmonary artery occlusive pressure and high cardiac output
b. High systemic vascular resistance and low cardiac output
c. Low pulmonary artery occlusive pressure and low cardiac output
d. Low systemic vascular resistance and high cardiac output

A

d. Low systemic vascular resistance and high cardiac output

As a consequence of the massive vasodilation associated with septic shock, in the early stages, cardiac output is high with low systemic vascular resistance. In septic shock, pulmonary artery occlusion pressure is not elevated. In the early stages of septic shock, systemic vascular resistance is low and cardiac output is high. In the early stages of septic shock, cardiac output is high

262
Q

A client is being discharged home after a large myocardial infarction and subsequent coronary artery bypass grafting surgery. The client’s sternal wound has not yet healed. What statement by the client most indicates a higher risk of developing sepsis after discharge?

a. All my friends and neighbors are planning a party for me.
b. I hope I can get my water turned back on when I get home.
c. I am going to have my daughter scoop the cat litter box.
d. My grandkids are so excited to have me coming home!

A

b. I hope I can get my water turned back on when I get home.

B ~ All these statements indicate a potential for leading to infection once the client gets back home. A large party might include individuals who are themselves ill and contagious. Having litter boxes in the home can expose the client to microbes that can lead to infection. Small children often have upper respiratory infections and poor hand hygiene that spread germs. However, the most worrisome statement is the lack of running water for handwashing and general hygiene and cleaning purposes.

263
Q

A client brought to the emergency department after a motor vehicle accident is suspected of having internal bleeding. Which question does the nurse ask to determine whether the client is in the early stages of hypovolemic shock?

a. “Are you more thirsty than normal?”
b. “When was the last time you urinated?”
c. “What is your normal heart rate?”
d. “Is your skin usually cool and pale?”

A

c. “What is your normal heart rate?”

The first manifestations of hypovolemic shock result from compensatory mechanisms. Signs of shock are first evident as changes in cardiovascular function. As shock progresses, changes in skin, respiration, and kidney function progress. The other questions would not identify early stages of shock.

264
Q

A client who has acidosis resulting from hypovolemic shock has been prescribed intravenous fluid replacement. Which fluid does the nurse prepare to administer?

a. Normal saline
b. Ringer’s lactate
c. 5% dextrose in water
d. 5% dextrose in 0.45% normal saline

A

b. Ringer’s lactate

Ringer’s lactate is an isotonic solution that acts as a volume expander. Also, the lactate acts as a buffer in the presence of acidosis. The other solutions do not contain any substance that would buffer or correct the client’s acidosis.

265
Q

The nurse is caring for a client who has hypovolemic shock. After administering oxygen, what is the priority intervention for this client?

a. Administer an aminoglycoside.
b. Initiate a dopamine hydrochloride (Intropin) drip.
c. Administer crystalloid fluids.
d. Initiate an intravenous heparin drip.

A

c. Administer crystalloid fluids.

IV therapy for fluid resuscitation is the primary intervention for hypovolemic shock. A dopamine hydrochloride drip is a secondary treatment if the client does not respond to fluids. Aminoglycosides and heparin are given to clients with septic shock.

266
Q

The nurse is administering prescribed sodium nitroprusside (Nipride) intravenously to a client who has shock. Which nursing intervention is a priority when administering this medication?

a. Ask if the client has chest pain every 30 minutes.
b. Assess the client’s blood pressure every 15 minutes.
c. Monitor the client’s urinary output every hour.
d. Observe the client’s extremities every 4 hours.

A

b. Assess the client’s blood pressure every 15 minutes.

The client receiving sodium nitroprusside should have his or her blood pressure assessed every 15 minutes. Higher doses can cause systemic vasodilation and can increase shock. The nurse should monitor the client’s pain, urinary output, and extremities, but these assessments do not directly relate to the nitroprusside infusion.